Preview (15 of 123 pages)

Preview Extract

ATI MENTAL HEALTH ACTUAL EXAM 304 QUESTIONS AND
CORRECT ANSWERS 2023-2024 UPDATE ALREADY A GRADED
WITH EXPERT GUIDED FEEDBACK|BRAND NEW!!
1. A nurse is caring for a client who has major depressive disorder. Which of the following
actions should the nurse take when developing a relationship with the client?
● Share personal information to help the client feel comfortable.
● Develop an emotional commitment to the client.
● Set boundaries with the client regarding personal space.
● Tell the client if she reminds the nurse of a personal friend or relative.
Answer: Set boundaries with the client regarding personal space.
Rationale:
Setting boundaries with the client regarding personal space is essential for maintaining a
professional therapeutic relationship. While it's important to establish rapport and trust,
sharing personal information can blur professional boundaries and may not be appropriate in
this context. Additionally, developing an emotional commitment to the client can lead to
burnout and may not be sustainable or beneficial for the client's overall care. Telling the client
if she reminds the nurse of a personal friend or relative may inadvertently influence the
client's perception and could compromise the therapeutic relationship.
2. A nurse is caring for a client who is in the acute manic phase of bipolar disorder. Which of
the following activities is appropriate for the nurse to suggest to the client? (the focus is on
safety and maintaining physical health)
● Attending a client's birthday party in the café
● Watching a movie with a group of clients in the day room
● Participating in a basketball game in the gym. (provide outlets for physical activity:
punching bags, physical exercise. Do not involve the client in activities that last a long time
or require a high level of concentration or detailed instruction)
● Walking with the nurse on the grounds of the facility. (Decrease stimulation without
isolating the client if possible. Be aware of noise, music, television, and other clients, all of
which can lead to an escalation of the client's behavior. In some cases, seclusion might be the
only way to safely decrease stimulation for the client)
Answer: Participating in a basketball game in the gym.
Rationale:

In the acute manic phase of bipolar disorder, it's crucial to provide outlets for physical
activity to help manage excess energy and reduce agitation. Activities like participating in a
basketball game in the gym offer a safe and structured environment for the client to release
energy and maintain physical health. This activity aligns with the goal of managing the
client's symptoms while ensuring safety.
3. A nurse is assisting in the care of a client who has schizophrenia and is experiencing a
hallucination. Which of the following actions by the nurse is appropriate?
● Act like the hallucination is real.
● Instruct the client to argue with the voices that are a part of the hallucination.(Symptom
management techniques include strategies such as using music to distract from “voices”,
attending activities, walking, talking to a trusted person when hallucinations are most
bothersome, and interacting with an auditory or visual hallucination by telling it to stop or go
away)
● Ask the client direct questions about the hallucination.(Ask the client directly about
hallucination. The nurse should not argue or agree with the clients view of the situation, but
can offer a comment, such as, “i don't hear anything but you seem to be feeling frightened”
● Tell the client that the hallucination is not a part of reality.
Answer: Ask the client direct questions about the hallucination.
Rationale:
Asking the client direct questions about the hallucination allows the nurse to assess the
client's current experience and provide appropriate support. It acknowledges the client's
reality without necessarily validating the hallucination as real. Providing reassurance and
offering grounding techniques can help the client manage distress related to the hallucination
while maintaining a therapeutic relationship.
4. A nurse is caring for a newly admitted client who is suspicious of the nursing staff and
other clients. Which of the following actions should the nurse take to establish a relationship
with the client
● Wait for the client to initiate interactions with the nurse.
● Set aside short, frequent times each day to spend with the client.
● Tell the client he reminds the nurse of her father.
● Disclose some personal information to the client.
Answer: Set aside short, frequent times each day to spend with the client.
Rationale:

Setting aside short, frequent times each day to spend with the client demonstrates the nurse's
commitment to building a therapeutic relationship and fostering trust. It allows the nurse to
gradually establish rapport with the client, despite initial suspicion or mistrust. Waiting for
the client to initiate interactions may prolong the process of relationship-building, while
disclosing personal information could blur professional boundaries and may not be
appropriate at this stage.
5. A nurse in an urgent care facility is caring for a client who has traumatic injuries following
an assault. The client sits quietly and calmly and tells the nurse “I'm fine”. The nurse should
recognize the client's behavior as which of the following reactions?
● Denial
● Displacement
● Projection
● Undoing
Answer: Denial
Rationale:
Denial is a defense mechanism commonly observed in individuals experiencing trauma or
distressing situations. The client's calm demeanor and statement of being fine despite evident
traumatic injuries suggest a psychological attempt to avoid or minimize the reality of the
situation. This behavior serves as a coping mechanism to protect the individual from
overwhelming emotions associated with the traumatic event.
6. A nurse is caring for a child who has autism spectrum disorder. Which of the following
findings should the nurse expect? SATA
● Short attention span
● Delayed language development
● Spins a toy repetitively
● Ritualistic behavior
● Consistent limit-testing
Answer: Short attention span, Delayed language development, Spins a toy repetitively,
Ritualistic behavior
Rationale:
Autism spectrum disorder (ASD) is characterized by a range of symptoms, including short
attention span, delayed language development, repetitive behaviors such as spinning toys, and
engaging in ritualistic behaviors. These behaviors are commonly observed in individuals with
ASD and are part of the diagnostic criteria.

7. A nurse is collecting data from a client who is taking chlorpromazine. Which of the
following findings should the nurse identify as extrapyramidal symptoms (EPS)? SATA
● Muscle contractions of the neck
● Fidgeting behavior
● Fluctuating vital signs
● Impaired gait
● Sexual dysfunction
Answer: Muscle contractions of the neck, Impaired gait
Rationale:
Extrapyramidal symptoms (EPS) are side effects associated with antipsychotic medications
like chlorpromazine. These symptoms include muscle contractions of the neck (torticollis or
dystonia) and impaired gait (parkinsonism). EPS can manifest as various movement disorders
and are important to monitor in clients taking antipsychotic medications.
8. A nurse is assisting in the care of a client who has bipolar disorder. The client states “i feel
like superman. I can do anything, I can fly home today and then become a U.S senator”
which of the following findings should the nurse document the client is exhibiting?
● Flight of ideas
● Grandiosity (Grandiose view of self and abilities. During manic phase)
● Reality testing
● Derealization
Answer: Grandiosity
Rationale:
The client's statement reflects grandiosity, which is a characteristic symptom of the manic
phase of bipolar disorder. Grandiosity involves an inflated sense of self-importance, abilities,
and achievements beyond what is realistic. This symptom often leads individuals with bipolar
disorder to engage in risky behaviors or make unrealistic plans due to their heightened
confidence and belief in their capabilities.
9. A nurse is caring for a client who has bipolar disorder and states that his latest project is
“revolutionizing the industry” . Which of the following behaviors is the client exhibiting?
● Flight of ideas
● Confabulation
● Clang associations
● Grandiosity (Grandiose view of self and abilities. During manic phase)
Answer: Grandiosity

Rationale:
The client's statement about revolutionizing the industry reflects grandiosity, a symptom
commonly observed in the manic phase of bipolar disorder. Grandiosity involves an
exaggerated sense of self-importance and abilities, leading the individual to believe they
possess exceptional talents or ideas. This symptom can contribute to impaired judgment and
risky decision-making during manic episodes.
10. A nurse is reinforcing teaching about valproate with a client who has bipolar disorder.
Which of the following information should the nurse include in the teaching?
● "Thyroid function tests must be performed every 6 months."
● "A pretreatment electroencephalogram (EEG) will be performed."
● "Liver function tests must be monitored regularly."
● "A white blood count must be monitored weekly."
Answer: Liver function tests must be monitored regularly.
Rationale:
Valproate is associated with hepatotoxicity, so it's important to monitor liver function tests
regularly to detect any signs of liver damage or dysfunction. This monitoring helps ensure the
safe use of valproate and allows for prompt intervention if adverse effects on liver function
occur. Thyroid function tests, EEG, and white blood count monitoring are not specific to
valproate and are not routinely performed as part of valproate therapy.
11. A nurse on an inpatient mental health unit is assisting with the admission of a client who
reports feeling depressed, sad, moody, and overly anxious. Which of the following
assessments should the nurse make first?
● Coping abilities
● Support systems
● Suicide risk
● Psychiatric history
Answer: Suicide risk
Rationale:
Assessing suicide risk is the priority when admitting a client reporting symptoms of
depression, sadness, and anxiety. Determining the client's risk of self-harm or suicide allows
for appropriate interventions to ensure the client's safety. Coping abilities, support systems,
and psychiatric history are important assessments but are secondary to immediate safety
concerns such as suicidal ideation or intent.

12. A nurse is discussing alcohol tolerance with a newly licensed nurse. Which of the
following statement by the newly licensed nurse indicates an understanding of the teaching?
● "A client who has alcohol tolerance develops physical changes when they haven't recently
ingested alcohol."
● "Alcohol tolerance causes the client to have an increased effect when taking opiates."
● "Alcohol tolerance can result in a decreased physical response to alcohol."
● "Alcohol tolerance is a medical emergency and can develop as a result of withdrawal."
Answer: "Alcohol tolerance can result in a decreased physical response to alcohol."
Rationale:
Alcohol tolerance refers to a decreased sensitivity to the effects of alcohol over time,
resulting in the need for higher doses to achieve the same effects. This decreased response to
alcohol can lead individuals to consume larger amounts of alcohol without experiencing the
expected intoxicating effects. Understanding alcohol tolerance is crucial for nurses in
assessing alcohol use and its potential consequences.
13. A nurse is contributing to the plan of care for a newly admitted client who has obsessivecompulsive disorder. Which of the following interventions should the nurse plan to include?
● Give negative reinforcement for ritualistic behavior.
● Provide a stimulating environment for the client.
● Provide a structured schedule of activities for the client.
● Limit time for rituals to 30 min each day.
Answer: Provide a structured schedule of activities for the client.
Rationale:
Providing a structured schedule of activities helps individuals with obsessive-compulsive
disorder (OCD) manage their symptoms by providing predictability and reducing anxiety. A
structured routine can help minimize the need for compulsive rituals and provide a sense of
control for the client. Negative reinforcement for ritualistic behavior is not recommended as it
may reinforce the behavior. A stimulating environment may exacerbate anxiety for clients
with OCD. Limiting time for rituals may increase anxiety and is not a recommended
intervention.
14. A client who has Rheumatoid arthritis shows her copper bracelet and says it helps
alleviate her pain when she wears it. Which of the following responses should the nurse
make?
● "Yes, I understand that you feel better wearing your bracelet."
● "Why do you think the copper helps with your arthritis?"

● "Believing objects have powers to make you feel better has no scientific basis."
● "I think you should rely more on your medication therapy than on your bracelet."
Answer: "Yes, I understand that you feel better wearing your bracelet."
Rationale:
Acknowledging the client's experience and understanding their perspective is important for
therapeutic communication. Validating the client's belief in the effectiveness of the copper
bracelet respects their autonomy and promotes trust in the nurse-client relationship.
Dismissing the client's belief or suggesting reliance on medication alone may undermine the
client's sense of control and autonomy in managing their pain.
15. A nurse is preparing to change the dressing on the lower leg of an older adult who is in a
wheelchair and has a history of maladaptive coping skills. The client begins swearing and
verbally abusing the nurse. Which of the following actions should the nurse take?
(always use the least restrictive action first)
● Tell the client when he will return and leave the room.
● Explain to the client why her behavior is inappropriate. Place wrist restraints on the client
to prevent psychomotor agitation.
● Move the client to a seclusion room.
Answer: Tell the client when he will return and leave the room.
Rationale:
The least restrictive action in response to verbal abuse is to calmly inform the client of when
the nurse will return and then leave the room. This approach removes the nurse from the
situation temporarily, allowing both the nurse and the client time to de-escalate. Placing wrist
restraints or moving the client to a seclusion room should only be considered as a last resort
when the client's behavior poses an immediate threat to themselves or others. Explaining to
the client why their behavior is inappropriate may escalate the situation further and is not the
most appropriate initial response.
16. As part of the plan of care for a client who has borderline personality disorder, the nurse
reviews the day schedule with him each morning. While doing so, the client states “why don't
you shut up already!” “I can read it myself, you know!” Which of the following is an
appropriate nursing response?
● "I know you can read it yourself, but will you?"
● "We do this every day. Why are you so angry with me this morning?"
● "I expect you to speak to me in a civil tone of voice."
● "Fine. Here is the schedule. I expect you to be on time for your therapy sessions."

Answer: "I expect you to speak to me in a civil tone of voice."
Rationale:
Setting limits on inappropriate behavior is crucial when working with clients with borderline
personality disorder. Responding assertively by expressing expectations for respectful
communication helps establish boundaries and maintains a therapeutic environment. This
response demonstrates consistency in enforcing behavioral expectations and reinforces
appropriate interaction.
17. A nurse is caring for a client who has depression and states she is too tired to get out of
bed or dress. Which of the following statements by the nurse is appropriate?
● "You really need to follow the rules of the unit and get out of bed."
● "If you do not get out of bed, you will not receive your meal."
● "I will help you sit up and get your slippers on."
● "You should rest in bed until you feel able to take part in unit activities."
Answer: "I will help you sit up and get your slippers on."
Rationale:
Offering assistance with activities of daily living in a supportive manner promotes the client's
sense of dignity and autonomy while addressing their physical needs. This response
acknowledges the client's current state and provides practical assistance without judgment or
coercion. Encouraging rest while also offering assistance when the client is ready aligns with
a client-centered approach to care.
18. A nurse is discussing legal exceptions to client confidentiality with nursing staff. Which
of the following statements by a staff member indicates an understanding?
● "The legal requirement for client confidentiality does not apply if the client is deceased."
● "Staff members are required to divulge information regarding a client's hospitalization to a
client's employer."
● "Health care workers can use client confidentiality for their own legal defense."
● "Providers are required to warn individuals if the client threatens harm."
Answer: "Providers are required to warn individuals if the client threatens harm."
Rationale:
The duty to warn is a legal exception to client confidentiality that requires healthcare
providers to inform potential victims or authorities if a client poses a risk of harm to
themselves or others. Understanding this exception is essential for protecting the safety of
individuals who may be at risk of harm from a client's actions.

19. A nurse in a mental health facility is caring for a client who becomes upset and breaks a
chair when a visitor does not arrive. The client remains agitated following initial attempts to
calm him down. Which of the following interventions should the nurse implement first?
● Restrain the client to prevent injury to himself or others.
● Place the client in a monitored seclusion room until he is calm.
● Offer the client an anti-anxiety medication.
● Plan with the client for how he can better handle frustration.
Answer: Plan with the client for how he can better handle frustration.
Rationale:
Collaborating with the client to develop coping strategies for managing frustration addresses
the underlying issue and empowers the client to develop healthier responses to stressors.
Restraint and seclusion should only be used as a last resort when there is an immediate risk of
harm to the client or others. Offering medication without addressing the underlying issue
does not promote long-term coping skills and may only provide temporary relief.
20. A nurse is caring for a client who has depression. After two days of treatment, the nurse
notices the client is suddenly more active and there are no longer signs of a depressive state.
Which of the following interventions should the nurse recommend for the plan of care?
● Encourage the family to take the client out of the facility for short periods of time.
● Reward the client for her change in behavior.
● Monitor the client's whereabouts at all times.
● Ask the client why her behavior has changed.
Answer: Monitor the client's whereabouts at all times.
Rationale:
Sudden changes in behavior, such as increased activity and loss of depressive symptoms,
could indicate a potential risk for suicide. Monitoring the client closely, especially in the
immediate period following a change in behavior, is essential for ensuring the client's safety.
Encouraging family outings, rewarding behavior, or asking the client about the change in
behavior may not address the potential underlying risk of suicide and may not be appropriate
interventions in this context.
21. A nurse on an outpatient unit is caring for a client who has major depressive disorder. The
nurse observes an improvement in the client's grooming when he comes to breakfast. Which
of the following statements should the nurse make?
● “You look very nice after your bathe."
● "You should do that more often. You look great!"

● "Everyone feels better after showering."
● "Why are you so dressed up? Is it a special occasion?"
Answer: “You look very nice after your bathe."
Rationale:
Providing positive reinforcement for the client's improved grooming acknowledges the effort
and promotes self-esteem. This statement offers a specific compliment related to the observed
behavior, which can encourage the client to continue engaging in self-care activities.
Avoiding generalized statements or questioning the client's motivation helps maintain a
supportive and non-judgmental approach.
22. A nurse in a long-term care facility is performing a mental status examination (MSE) for a
newly admitted client who has dementia. Which of the following data should the nurse
include? SATA
● Grooming
● Long-term memory
● Support systems
● Affect
● Presence of pain
Answer: Grooming, Long-term memory, Affect
Rationale:
In a mental status examination (MSE), assessment of grooming provides information about
the client's self-care abilities and overall functioning. Long-term memory assessment helps
evaluate cognitive function and can reveal deficits associated with dementia. Affect
assessment includes observation of the client's appearance, behavior, speech, and mood,
which can provide insight into their emotional state and cognitive functioning. While support
systems and presence of pain are important aspects of a comprehensive assessment, they are
not typically included in a standard MSE.
23. A nurse in an assisted living facility is caring for a client in the early stages of dementia.
The client has been oriented to name and place and is usually cooperative. Which of the
following nursing actions is appropriate if the client refuses to take her medication?
● Crush the pills, if not contraindicated, and hide them in applesauce.
● Ask the client to express her reasons for refusing the morning medications and document
the event.
● Try to talk the client into adherence by telling her the possible implications of missing a
dose.

● Notify the charge nurse of the need for evaluation of the client's level of competence.
Answer: Ask the client to express her reasons for refusing the morning medications and
document the event.
Rationale:
Encouraging the client to express her reasons for refusing medication promotes autonomy
and allows the nurse to address underlying concerns or barriers to adherence. Documenting
the event ensures that the refusal and any pertinent information are recorded for future
reference and communication with other healthcare providers. Crushing pills without consent
may violate the client's rights and is not appropriate without further exploration of the refusal.
Involving the charge nurse for evaluation of competence may be necessary if the client's
refusal raises concerns about capacity, but it should not be the first action taken.
24. A nurse is caring for a client who has moderate anxiety disorder. The client is pacing in
the hallway and states “I am at the end of my rope, I can't take it anymore”. Which of the
following responses should the nurse make?
● "Most clients with anxiety issues benefit from lying down."
● "Walk with me to an area where we can talk about how you're feeling."
● "Providers usually recommend relaxation exercises for clients who are as upset as you are."
● "An anti-anxiety pill works best for situations like this. Let me get you one."
Answer: "Walk with me to an area where we can talk about how you're feeling."
Rationale:
Offering to walk with the client to a quieter area for discussion provides an opportunity for
therapeutic communication and emotional support. Moving to a less stimulating environment
can help reduce the client's anxiety and facilitate open dialogue about their feelings and
concerns. Providing immediate pharmacological intervention may be appropriate in some
cases but should not be the first response unless the client's safety is at risk.
25. A nurse is caring for a client who has right-sided paralysis secondary to a stroke. The
client's adult son states to the nurse “None of this would have happened if I was here.” Which
of the following responses should the nurse make?
● "It seems that you feel responsible for what happened to your mother."
● "Your mother will be fine. I wouldn't worry so much."
● "Let's talk about how your mother's therapy sessions are going."
● "Why do you feel responsible for your mother's illness?"
Answer: "It seems that you feel responsible for what happened to your mother."
Rationale:

Reflecting the client's statement back to him acknowledges his feelings and provides an
opportunity for further exploration and expression. This therapeutic technique encourages the
client to clarify his emotions and promotes understanding and empathy. Avoiding dismissive
statements or redirecting the conversation helps maintain a supportive and empathetic
approach to communication.
26. A nurse is caring for a group of clients in a mental health facility. Which of the following
clients recommend the provider see first?
● A client taking clozapine who has a sore throat and mild fever.
● A client taking chlorpromazine who is napping frequently throughout the day
● A client taking risperidone who has gained 5 lb in 3 weeks.
● A client taking olanzapine who experiences dizziness upon standing
Answer: A client taking clozapine who has a sore throat and mild fever.
Rationale:
Clozapine, an atypical antipsychotic medication, can cause agranulocytosis, a potentially lifethreatening condition characterized by a severe decrease in white blood cells. Symptoms such
as a sore throat and fever may indicate infection, which can be a sign of agranulocytosis in
clients taking clozapine. Prompt evaluation by the provider is essential to assess for
agranulocytosis and initiate appropriate treatment. Monitoring for agranulocytosis is critical
in clients taking clozapine due to the risk of serious infection.
27. A nurse in a community clinic is caring for a 20-month-old toddler who has spiral
fractures of the right ulna and radius. Which of the following findings should the nurse
recognize as a potential indication of abuse?
● The child begins to cry when her arm is examined by the provider.
● The child's examination shows a single injury.
● The child was brought to the facility 30 min after the injury occurred.
● The parents report that the child injured herself by falling off the couch.
Answer: The parents report that the child injured herself by falling off the couch.
Rationale:
In cases of suspected child abuse, inconsistent or implausible explanations for injuries
provided by caregivers raise significant concern. The fact that the injury occurred during a
seemingly innocuous event, such as falling off a couch, may indicate an attempt to conceal
abuse. Additionally, the presence of spiral fractures, which are often associated with twisting
injuries, can be suggestive of non-accidental trauma. Prompt assessment and reporting of
suspected child abuse are essential to ensure the safety and well-being of the child.

28. A nurse is discussing suicide prevention with nursing staff. Which of the following
actions should the nurse identify as an example of secondary intervention?
● Providing support for family and friends following a suicide
● Identifying individuals who are at higher risk for attempting suicide
● Recognizing the warning signs of suicide
● Performing life-saving measures following a suicide attempt
Answer: Identifying individuals who are at higher risk for attempting suicide
Rationale:
Secondary intervention involves identifying and intervening with individuals who are at
higher risk for suicide before a suicide attempt occurs. This includes implementing
preventive measures and providing targeted interventions to mitigate risk factors and promote
protective factors. Secondary prevention aims to reduce the incidence and impact of suicidal
behavior by intervening early in the trajectory toward suicide risk.
29. A nurse is caring for a client in the emergency department who has had a traumatic
amputation of his left arm in an industrial accident 1 hour ago. The nurse should expect the
client to be in which of the following stages of grief?
● Bargaining
● Depression
● Denial
● Acceptance
Answer: Denial
Rationale:
Denial is a common initial response to traumatic events or significant losses, such as
amputation. The client may have difficulty accepting the reality of the situation and may
exhibit denial as a coping mechanism to protect themselves from the overwhelming emotions
associated with the loss. It is typical for individuals to initially deny the severity or
permanence of their situation before progressing through the stages of grief.
30. A nurse is caring for a client who has a new diagnosis of HIV. The client states “I don't
care what the doctor says, there is no way I can have HIV, I don't need treatment for
something I don't have”. The nurse should identify the client is experiencing which of the
following crises?
● Adventitious
● Internal
● Maturational

● Situational
Answer: Situational
Rationale:
Situational crises are unexpected events that disrupt an individual's regular life patterns and
require significant adjustment. A new diagnosis of HIV can be considered a situational crisis
as it represents a sudden and unexpected change that challenges the individual's beliefs,
coping mechanisms, and future plans. The client's denial of the diagnosis and refusal of
treatment reflect their struggle to accept and adapt to the new reality, which is characteristic
of a situational crisis.
31. A nurse is observing a newly licensed nurse provide care for a client who has bipolar
disorder and is experiencing mania. The client continually runs around the unit asking people
to dance with her. Which of the following actions by the newly licensed nurse is appropriate?
● Turns on a dance video so the client can burn off excess energy.
● Offers the client a low-calorie snack if she stops the behavior.
● Asks the client to go outside with him and sit in the garden area.
● Observes the client closely for the development of aggressive behavior.
Answer: Observes the client closely for the development of aggressive behavior.
Rationale:
In clients experiencing mania, excessive physical activity and impulsivity may escalate into
aggressive behavior. Therefore, closely observing the client for signs of agitation or
aggression is crucial for ensuring the safety of both the client and others. Interventions should
focus on preventing potential harm and maintaining a therapeutic environment. While
engaging in physical activity or offering a snack may temporarily redirect the client's
behavior, these interventions may not address the underlying issues associated with mania or
prevent the potential escalation of agitation.
32. A nurse is assisting in a group therapy meeting and is sharing a humorous story unrelated
to anyone in the group. When the group laughs at the story, A client with schizophrenia jumps
up and runs out while yelling “You are all making fun of me!”. Which of the following
behaviors is this client displaying?
● Grandeur
● Somatic delusion
● Erotomania
● Ideas of reference
Answer: Ideas of reference

Rationale:
Ideas of reference involve misconstruing trivial events and attaching personal significance to
them. In this scenario, the client with schizophrenia misinterprets the laughter in the group as
directed towards them, despite the humorous story being unrelated to the client. The client's
reaction demonstrates a belief that others are making fun of them, reflecting ideas of
reference characteristic of schizophrenia. This misinterpretation can contribute to feelings of
persecution or paranoia commonly experienced by individuals with schizophrenia.
33. A nurse is assisting in the care of an adolescent client who has a new diagnosis of
schizophrenia. The client's parents are tearful and express feelings of guilt. Which of the
following responses should the nurse make?
● Let's talk about what is causing you to feel guilty."
● "You should not feel guilty about your daughter's diagnosis since schizophrenia is not
preventable."
● "Everything will be alright once she receives the proper treatment."
● "Why do you feel guilty about your daughter's diagnosis?"
Answer: "Let's talk about what is causing you to feel guilty."
Rationale:
Offering the opportunity for the parents to express their feelings of guilt allows for open
communication and exploration of their emotions. Validating their experiences and providing
a supportive environment for expression can help alleviate their distress and foster
understanding. Addressing the underlying causes of guilt enables the nurse to offer
appropriate support and education to the parents, facilitating their adjustment to their
daughter's diagnosis and treatment.
34. A nurse is reviewing the admission laboratory values for a client who has a history of
bulimia nervosa. Which of the following findings is the nurse's priority?
● Serum chloride 96 mEq/L
● Potassium 2.8 mEq/L
● Hgb 11g/dL
● Serum amylase 240 units/L
Answer: Potassium 2.8 mEq/L
Rationale:
Hypokalemia (low potassium) is a potentially life-threatening complication of bulimia
nervosa due to recurrent vomiting and laxative abuse. Hypokalemia can lead to cardiac
dysrhythmias and other serious complications. Therefore, correcting potassium imbalances is

a priority in the management of clients with bulimia nervosa. Close monitoring and prompt
intervention are essential to prevent adverse outcomes associated with electrolyte
disturbances.
35. A nurse is caring for a client who has alcohol use disorder and is experiencing
withdrawal. Which of the following interventions is the nurse's priority?
● Prevent the client from harming herself or others.
● Support the client's coping skills.
● Identify the use of defense mechanisms.
● Develop goals for treatment.
Answer: Prevent the client from harming herself or others.
Rationale:
During alcohol withdrawal, clients are at risk for experiencing severe symptoms such as
delirium tremens, seizures, and hallucinations, which can result in harm to themselves or
others. Therefore, the priority intervention is to ensure the client's safety by preventing harm.
This may involve close monitoring, implementing safety precautions, and providing
supportive care. Once the client's safety is ensured, other interventions, such as supporting
coping skills and developing treatment goals, can be addressed.
36. A nurse is reinforcing teaching with a client about Alcoholics Anonymous (AA). Which
of the following statements by the client indicates an understanding of the program's basic
concepts?
● "I am responsible for my alcoholism."
● "I need to identify things that cause me to be an alcoholic."
● "I am powerless over my addiction to alcohol."
● "I need to see a counselor who will be responsible for my recovery."
Answer: "I am powerless over my addiction to alcohol."
Rationale:
The statement "I am powerless over my addiction to alcohol" reflects one of the fundamental
principles of Alcoholics Anonymous (AA), which emphasizes the recognition of
powerlessness over alcohol and the need for surrendering to a higher power for recovery. This
acknowledgment is a key step in the twelve-step program and is essential for individuals
seeking recovery from alcohol use disorder. It reflects an understanding of the program's
philosophy and approach to overcoming addiction.

37. A nurse is caring for a client who escapes anxiety-causing thoughts by ignoring their
existence. The nurse should recognize his behavior as which of the following defense
mechanisms?
● Repression
● Splitting
● Sublimation
● Undoing
Answer: Repression
Rationale:
Repression involves unconsciously putting unacceptable ideas, thoughts, and emotions out of
awareness. In this scenario, the client copes with anxiety-causing thoughts by pushing them
out of consciousness, effectively ignoring their existence. Repression allows the individual to
avoid facing uncomfortable or distressing thoughts or feelings, thereby reducing anxiety in
the short term.
38. A nurse in a mental health facility is discussing antidepressant medications with a newly
licensed nurse, comparing Selective serotonin reuptake inhibitors (SSRIs) and Tricyclic
antidepressants (TCAs). Which of the following information should the nurse include about
TCAs?
● Less effective in relieving depressive symptoms
● Low probability of causing sedation
● More likely to be prescribed as initial treatment
● Increased risk of cardiovascular adverse effects
Answer: Increased risk of cardiovascular adverse effects
Rationale:
Tricyclic antidepressants (TCAs) are associated with an increased risk of cardiovascular
adverse effects, such as orthostatic hypotension, tachycardia, and cardiac conduction
abnormalities. This risk is particularly relevant in clients with pre-existing cardiovascular
conditions. TCAs are less commonly prescribed as initial treatment for depression compared
to SSRIs due to their side effect profile and potential for overdose toxicity.
39. A charge nurse overhears another nurse talking with a client who has schizophrenia.
Suddenly the client yells, "I am the devil! I am God! Open the gate for me!" Which of the
following responses by the nurse requires a charge nurse to intervene?
● “Tell me who you are.”
● “I don't understand. Can you tell me what that means?”

● “Are you saying that you are both good and bad?”
● “There is no gate.”
Answer: “There is no gate.”
Rationale:
Responding with "There is no gate" can be perceived as invalidating the client's experiences
and may exacerbate their distress. It dismisses the client's beliefs without offering
understanding or validation, potentially worsening the therapeutic relationship and escalating
the client's agitation. A more therapeutic response would involve acknowledging the client's
feelings and experiences while redirecting the conversation to address the underlying
emotions or concerns.
40. A nurse in an acute care facility is caring for a preschool-age child who is terminally ill
and receiving palliative care. The parents tell the nurse that they have decided to take their
child home. Which of the following responses should the nurse make?
● "Tell me why you want to take your child home."
● "The pediatrician is the only one who can discharge your child."
● "You have not given the medication a chance to work."
● "I will help you get ready to leave."
Answer: "Tell me why you want to take your child home."
Rationale:
Asking the parents why they want to take their child home allows the nurse to explore their
reasons and concerns. It demonstrates empathy and respect for the parents' wishes while
providing an opportunity for open communication. Understanding the parents' motivations
can help the nurse address any misconceptions, fears, or unmet needs, and provide
appropriate support and guidance. Additionally, it facilitates collaborative decision-making
and ensures that the child's best interests are considered.
41. A nurse in a long-term care setting is caring for a client who has Alzheimer’s disease. The
client states, “I just came back from a hard day's work in my office”. The nurse should
recognize this statement is an example of which of the following coping mechanisms?
● Perseveration
● Confabulation
● Thought deletion
● Tangentiality
Answer: Confabulation
Rationale:

Confabulation is the fabrication of false or distorted memories or narratives to fill in memory
gaps. Clients with Alzheimer's disease often experience memory deficits and may confabulate
to make sense of their surroundings or experiences. In this scenario, the client's statement
about coming back from work is likely a confabulation, as it reflects a false memory or belief.
42. A home care nurse is visiting an older adult client who tells the nurse that she is feeling
tired, is unable to shop for groceries, and would like the nurse to shop for her. Shopping and
performing personal errands for the client are prohibited in the nurse's job description. Which
of the following is an appropriate nursing response?
● "I won't be able to shop for you today because I have to get home to my family."
● "I would be happy to do whatever I can to help you."
● "What I think you should do is wait for the days when you feel better and do your grocery
shopping then."
● "Let's look at some other resources to solve this problem."
Answer: "Let's look at some other resources to solve this problem."
Rationale:
This response acknowledges the client's request while redirecting the conversation to explore
alternative resources or solutions. It demonstrates the nurse's willingness to assist the client
within the scope of nursing practice while respecting professional boundaries. Collaborating
with the client to identify available community resources or support services promotes
autonomy and empowers the client to address her needs effectively.
43. A nurse in a community clinic is speaking to other nurses about risk factors for domestic
violence. Which of the following statements by one of the other nurses indicates the need for
further teaching?
● "I have heard that abusers try to keep their partner isolated from others."
● "I know that abusers lack social support and social skills."
● "I know that men who are abusers gain power through intimidation."
● "I have heard that abusers think of themselves as important and have high self-esteem."
Answer: "I know that abusers lack social support and social skills."
Rationale:
This statement incorrectly attributes lack of social support and social skills to abusers, which
is not necessarily true. Abusers may have social support networks and varying levels of social
skills. Understanding risk factors for domestic violence requires recognizing patterns of
behavior, power dynamics, and societal factors rather than making assumptions about
individual characteristics.

44. A nurse is caring for a client who is experiencing acute anxiety. Which of the following
actions should the nurse take? SATA
● Avoid eye contact when addressing the client.
● Establish rapport with the client.
● Identify the cause of the anxiety.
● Validate the client's feelings.
● Speak to the client using a high-pitched voice.
Answer: Establish rapport with the client.
Identify the cause of the anxiety.
Validate the client's feelings.
Rationale:
When caring for a client experiencing acute anxiety, it's important to establish rapport to
build trust and create a supportive environment. Identifying the cause of the anxiety can help
address the underlying issue and provide appropriate interventions. Validating the client's
feelings demonstrates empathy and understanding, which can help alleviate distress. Avoiding
eye contact and speaking with a high-pitched voice may increase the client's anxiety and
should be avoided.
45. A nurse is assisting in the plan of care for a client who has a personality disorder and
demonstrates manipulative behavior. Which of the following interventions is appropriate to
include in the plan of care?
● Allow manipulation so as to not raise the client's anxiety.
● Create a strict schedule for the client's activities to discourage manipulation.
● Institute consequences for manipulative behavior.
● Bargain with the client to discourage manipulative behavior.
Answer: Institute consequences for manipulative behavior.
Rationale:
Manipulative behavior should not be reinforced or tolerated, as it can hinder therapeutic
progress and disrupt the therapeutic environment. Instituting consequences for manipulative
behavior helps establish boundaries and accountability, promoting healthier coping
mechanisms and interpersonal skills. Consistent enforcement of consequences encourages the
client to take responsibility for their actions and encourages more adaptive behaviors.
46. A nurse is reinforcing discharge teaching with a client who has bipolar disorder and is to
begin taking lithium. Which of the following instructions should the nurse include in the
teaching?

● "Take the medication on an empty stomach."
● "Avoid dieting."
● "Decrease sodium intake."
● "Limit daily fluid intake to less than 1200 milliliters."
Answer: "Decrease sodium intake."
Rationale:
Lithium levels can be affected by sodium levels in the body. High sodium levels can lead to
decreased lithium levels, potentially reducing the effectiveness of the medication. Therefore,
clients taking lithium should be advised to maintain a consistent, moderate sodium intake and
avoid excessive dietary sodium. Decreasing sodium intake helps stabilize lithium levels and
optimize therapeutic outcomes.
47. A nurse is caring for a client who has schizophrenia and is taking haloperidol. The nurse
should monitor the client for which of the following adverse effects?
● Increased salivation
● Serotonin syndrome
● Tardive dyskinesia
● Increased menstrual bleeding
Answer: Tardive dyskinesia
Rationale:
Haloperidol, a first-generation antipsychotic, is associated with a risk of developing tardive
dyskinesia, a movement disorder characterized by involuntary, repetitive movements of the
face, tongue, and extremities. Monitoring for signs of tardive dyskinesia, such as facial
grimacing, lip smacking, and choreiform movements, is essential during antipsychotic
therapy. Increased salivation is not typically associated with haloperidol use, and serotonin
syndrome is more commonly associated with serotonergic medications.
48. A nurse is contributing to the plan of care for a client who has signs of alcohol
intoxication. Which of the following interventions should the nurse include in the client's plan
of care? SATA
● Obtain a blood sample.
● Prepare the client for a CT scan.
● Check the client's pupil reactivity.
● Obtain a prescription for as-needed restraints
● Perform a developmental screening test.
Answer: Obtain a blood sample.

Check the client's pupil reactivity.
Rationale:
When caring for a client with signs of alcohol intoxication, it's important to assess their
physiological status and rule out any potential medical complications. Obtaining a blood
sample allows for the assessment of blood alcohol levels, which can guide treatment
decisions. Checking the client's pupil reactivity helps evaluate neurological function and
detect signs of central nervous system depression. While developmental screening tests may
be important in other contexts, they are not relevant to the immediate care of a client with
alcohol intoxication.
49. A nurse is discussing comorbidities associated with eating disorders with a newly licensed
nurse. Which of the following comorbidities should the nurse include in the discussion?
● Anxiety
● Obsessive-compulsive disorder
● Schizophrenia
● Breathing-related sleep disorder
● Depression
Answer: Anxiety
Obsessive-compulsive disorder
Depression
Rationale:
Eating disorders frequently co-occur with other psychiatric conditions, including anxiety
disorders, obsessive-compulsive disorder (OCD), and depression. These comorbidities can
exacerbate eating disorder symptoms and complicate treatment. Addressing both the eating
disorder and any co-occurring psychiatric conditions is essential for comprehensive and
effective management.
50. A nurse is collecting data from a client who reports that he has obsessive-compulsive
disorder (OCD). Which of the following findings should the nurse expect?
● Difficulty relaxing
● Irrational fear of certain objects
● Rule-conscious behavior
● Unaware of compulsions
● Perfectionist behavior
Answer: Irrational fear of certain objects
Rule-conscious behavior

Perfectionist behavior
Rationale:
Obsessive-compulsive disorder (OCD) is characterized by the presence of obsessions
(persistent, intrusive thoughts or urges) and/or compulsions (repetitive behaviors or mental
acts performed in response to obsessions). Clients with OCD often experience irrational fears
or concerns, exhibit rule-conscious behavior, and engage in perfectionist tendencies.
Difficulty relaxing may occur as a result of anxiety related to OCD, and clients are typically
aware of their compulsions, although they may find them difficult to control.
51. A nurse is collecting data from a client who has histrionic personality disorder. Which of
the following findings should the nurse expect?
● Seductive
● Grandiose
● Callous toward others
● Preoccupied with details
Answer: Seductive
Rationale:
Histrionic personality disorder is characterized by attention-seeking behavior, dramatic
expression of emotions, and a need to be the center of attention. Clients with histrionic
personality disorder often exhibit seductive or provocative behavior to gain attention or
validation from others.
52. A nurse is assisting in the care of a client who has schizophrenia. The client states,” the
government is forcing thoughts into my brain through satellites”. The nurse should document
that the client is experiencing which of the following types of delusions?
● Persecution
● Control
● Erotomanic
● Somatic
Answer: Control
Rationale:
The client's belief that external forces, such as the government or satellites, are controlling or
implanting thoughts into their mind represents a delusion of control. This type of delusion
involves the false belief that one's thoughts, feelings, or actions are being manipulated by an
external source.

53. A nurse is caring for a client following the recent, sudden death of his partner. The client
says,” I feel paralyzed and can’t seem to cope with work or family responsibility anymore”.
Which of the following types of crisis is the client demonstrating?
● Situational
● Maturational
● Adventitious
● Developmental
Answer: Situational
Rationale:
The client's response to the sudden death of his partner is consistent with a situational crisis,
which arises from unexpected life events or circumstances. Situational crises disrupt an
individual's coping mechanisms and require adaptive strategies to manage the resulting
stressors.
54. A nurse is assisting in collecting data from a client who has acute phencyclidine (PCP)
intoxication. Which of the following findings should the nurse expect?
● Bradycardia
● Paranoia
● Hypoglycemia
● Hyperphagia
Answer: Paranoia
Rationale:
Phencyclidine (PCP) intoxication commonly presents with symptoms such as paranoia,
hallucinations, agitation, and disorientation. Paranoia, or extreme suspicion or distrust of
others, is a hallmark feature of PCP intoxication and can contribute to aggressive or violent
behavior in affected individuals.
55. A nurse is caring for a client who is experiencing a panic attack. Which of the following
actions is the nurse's priority?
● Offer the client high-calorie fluids.
● Remain with the client in a quiet area.
● Administer an antianxiety medication to the client.
● Teach the client relaxation exercises.
Answer: Remain with the client in a quiet area.
Rationale:

During a panic attack, the client may feel overwhelmed by intense fear or discomfort.
Remaining with the client in a calm, supportive manner and providing a quiet environment
can help reduce stimulation and promote a sense of safety and security. This intervention
helps prevent escalation of anxiety and provides reassurance to the client.
56. A nurse is caring for a client following major spinal surgery who is reporting pain. The
client's partner tells the nurse,” I wish I could do something to make my wife feel better”.
Which of the following responses to the nurse make?
● "I wish there was more that I could do to relieve your wife's pain, too."
● "We're doing everything we can to keep your wife comfortable."
● "It must be very difficult for you to see your wife in pain."
● "I'm sure your wife will begin to feel better soon."
Answer: "It must be very difficult for you to see your wife in pain."
Rationale:
This response acknowledges the partner's concern and empathizes with their distress over
witnessing their loved one in pain. It validates the partner's feelings and demonstrates
understanding of the emotional impact of the situation. Offering empathy and support to the
partner fosters a therapeutic relationship and enhances communication and collaboration in
the client's care.
57. A nurse is caring for a client who has generalized anxiety disorder (GAD). The client is
consistently late for appointments and ignores household chores. The client states, I’m just
too stressed. I need to be taken care of”. The nurse should identify this behavior as the
maladaptive use of which of the following defense mechanisms?
● Dissociation
● Introjection
● Regression
● Repression
Answer: Regression
Rationale:
Regression involves reverting to an earlier stage of development or adopting immature
behaviors as a means of coping with stress or anxiety. In this scenario, the client's statement
and behavior suggest a desire to avoid responsibility and revert to a more dependent,
childlike state to alleviate stress. This represents a maladaptive coping mechanism
characteristic of regression.

58. A nurse is caring for a client who has depression and is taking a monoamine oxidase
inhibitor (MAOi). The nurse should inform the client that his diet may include which of the
following foods?
● Cheddar cheese and sourdough bread
● Cottage cheese and bananas
● Beer and red wine
● Corned beef and sauerkraut
Answer: Cottage cheese and bananas
Rationale:
Clients taking monoamine oxidase inhibitors (MAOIs) should avoid foods high in tyramine,
as consuming these foods can lead to a hypertensive crisis. Cottage cheese and bananas are
low in tyramine and are generally considered safe to consume while taking MAOIs. Foods
such as cheddar cheese, sourdough bread, beer, red wine, and sauerkraut are high in tyramine
and should be avoided.
59. A nurse is caring for an older adult client admitted to a hospital following a cerebral
vascular accident. The Client's daughter tells the nurse” I wish I could stay with my father but
I need to go home to my family”. Which of the following responses to the nurse make?
● "You are feeling drawn in two different directions."
● "Don't worry. We'll take good care of your father while you are gone."
● "Perhaps you could stay here and just call your family to see how they are doing."
● "There's nothing you can do here. You should go home to your family."
Answer: "You are feeling drawn in two different directions."
Rationale:
This response validates the daughter's conflicting feelings and acknowledges the difficulty of
her situation. It reflects empathy and understanding of the daughter's emotional struggle,
which can help facilitate communication and support between the nurse and the client's
family. Additionally, it normalizes the daughter's experience of conflicting responsibilities
and emotions.
60. A nurse is assessing a young adult client who has a new diagnosis of idiopathic juvenile
arthritis. The client states,” the pain in my joints is just a temporary thing. If I keep eating
right and exercising it’ll go away”. The nurse should identify that the client is using which of
the following defense mechanisms?
● Denial
● Displacement

● Rationalization
Answer: Denial
Rationale:
Denial involves refusing to acknowledge the existence of a painful or anxiety-provoking
reality. In this scenario, the client's statement suggests a refusal to accept the chronic nature
of idiopathic juvenile arthritis and instead minimizes the severity of the condition by
attributing the pain to temporary factors such as diet and exercise. Denial can serve as a
defense mechanism to protect the individual from the emotional distress associated with the
diagnosis.
61. A nurse on the mental health unit is assisting with several group therapy sessions. The
nurse should include which of the following information about using group therapy as a
treatment method?
● "It establishes a situation where the client can relate to others and share perceptions."
● "It enables clients to see that other individuals have mental health issues."
● "It is economical, since one staff member can treat many clients at once."
● "It provides a forum to reinforce client teaching regarding medication administration."
Answer: "It establishes a situation where the client can relate to others and share
perceptions."
Rationale:
Group therapy provides a supportive environment where clients can interact with others who
have similar experiences or challenges. It fosters a sense of belonging, encourages peer
support, and allows clients to share their thoughts, feelings, and coping strategies. Through
group therapy, clients can gain insight into their own behaviors and emotions while learning
from the experiences of others.
62. A nurse is caring for a client who has schizophrenia. The nurse observes that the client
consistently does the opposite of what is told. The nurse recognizes this as which of the
following alterations in behavior?
● Automatic obedience
● Waxy flexibility
● Negativism
● Impaired impulse control
Answer: Negativism
Rationale:

Negativism is a characteristic behavior observed in some clients with schizophrenia where
they resist or oppose instructions or suggestions. They may do the opposite of what is
requested or exhibit passive resistance. This behavior can be a manifestation of the client's
disorganized or impaired thought processes.
63. A nurse in an acute care mental health facility is sitting with a client with schizophrenia.
The client whispers to the nurse,” I’m being kept in this prison against my will. Please try to
get me out”. Which of the following responses to the nurse make?
● "Why do you feel that you need to leave?”
● "You feel that you don't belong here?"
● "We are here to help you and give you the care that you need right now."
● "Try to take some deep breaths and I'm sure you'll feel better."
Answer: "We are here to help you and give you the care that you need right now."
Rationale:
This response acknowledges the client's concerns and provides reassurance that the staff is
committed to their well-being and safety. It validates the client's feelings without confirming
or denying the delusional belief. By focusing on the present and offering support, the nurse
helps establish a therapeutic relationship and promotes trust and rapport with the client.
64. A nurse is caring for a client who is experiencing a manic episode. Other clients begin to
complain about her disruptive behavior on the unit. Which of the following nursing
interventions is appropriate?
● Warn the client that further disruptions will result in seclusion.
● Ignore the client's behavior, realizing that it is consistent with her illness.
● Set limits on the client's behavior and be consistent in approach.
● Ask the client to recommend consequences for disruptive behavior.
Answer: Set limits on the client's behavior and be consistent in approach.
Rationale:
Setting limits on the client's behavior and enforcing unit rules consistently is essential for
maintaining a therapeutic environment and ensuring the safety and well-being of all clients.
While the client's behavior may be a manifestation of her manic episode, it is important to
address disruptive behavior promptly and establish boundaries to prevent escalation and
maintain order on the unit.
65. A nurse is assisting the charge nurse to prepare for an in-service with the nursing staff to
discuss substance use disorders. Which of the following should the nurse include as an
etiological factor of addictive disorders?

● Poor educational resources
● Low self-esteem
● Family history of addiction
● Lack of support system
● Asian ethnicity
Answer: Family history of addiction
Rationale:
A family history of addiction is recognized as a significant risk factor for the development of
addictive disorders. Genetic predisposition and environmental influences within the family
contribute to the likelihood of substance abuse and addiction. Individuals with a family
history of addiction may have a higher susceptibility to developing substance use disorders
due to genetic and familial influences on behavior and substance use patterns.
66. A nurse is caring for a client who has anorexia nervosa and insists on exercising 3 times a
day. Which of the following actions should the nurse take?
● Allow the client to exercise once per day for a set amount of time.
● Allow the client to exercise when she wants as long as she eats 50% of all meals.
● Remind the client that if her weight decreases, she will lose a privilege.
● Ask the client why she feels the need to exercise so often.
Answer: Allow the client to exercise once per day for a set amount of time.
Rationale:
An essential aspect of caring for clients with anorexia nervosa is establishing structured meal
plans and exercise regimens to promote healthy behaviors and prevent excessive exercise that
can exacerbate weight loss and contribute to physical complications. Allowing the client to
exercise once per day for a set amount of time under supervision helps balance physical
activity with nutritional intake and supports the client's recovery while minimizing the risk of
harm.
67. A nurse is contributing to the plan of care for a client who has dementia. Which of the
following actions should the nurse include in the plan of care?
● Post a written schedule of daily activities.
● Provide a consistent daily routine.
● Allow the client to choose free-time activities.
Answer: Provide a consistent daily routine.
Rationale:

Consistency and routine are crucial components of care for clients with dementia as they
provide predictability and structure, which can help reduce confusion, anxiety, and agitation.
A consistent daily routine helps the client feel more secure and comfortable in their
environment, enhances their sense of control, and promotes engagement in meaningful
activities. Providing a structured daily schedule supports cognitive function and
independence while facilitating successful participation in daily tasks and activities.
68. A nurse in an acute care mental health facility is contributing to the plan of care of a client
who is newly diagnosed with schizophrenia and is verbalizing paranoid delusions. Which of
the following interventions should the nurse include in the plan?
● Schedule a variety of competitive, stimulating group activities for the client.
● Set limits on the amount of time the client talks about delusions.
● Avoid asking the client about triggers for the delusions.
● Tell the client that the delusions are not real.
Answer: Set limits on the amount of time the client talks about delusions.
Rationale:
Setting limits on the amount of time the client spends discussing delusions helps maintain a
therapeutic focus during interactions and prevents excessive reinforcement or elaboration of
paranoid beliefs. While acknowledging the client's experiences and emotions is important,
redirecting conversations away from delusional content toward more reality-based topics can
help mitigate distress and support engagement in meaningful activities. Limiting discussions
about delusions also helps maintain boundaries and fosters a therapeutic environment
conducive to treatment and recovery.
69. A nurse is reviewing laboratory results for a client who is receiving lithium carbonate.
The client's blood lithium level is 1.8 mEq/L. Which of the following actions should the
nurse take?
● Administer the medication as scheduled.
● Withhold the medication.
● Repeat the lithium level test in 12 hr.
● Administer half a dose at the next scheduled time.
Answer: Withhold the medication
Rationale:
A blood lithium level of 1.8 mEq/L is above the therapeutic range and indicates potential
lithium toxicity. It is essential to withhold further doses of lithium and notify the healthcare
provider immediately for further evaluation and instructions. Lithium toxicity can lead to

serious adverse effects, including neurological symptoms, renal impairment, and cardiac
arrhythmias. Repeat testing and dose adjustments may be necessary to ensure the client's
safety and maintain lithium levels within the therapeutic range.
70. A nurse is caring for a young adult client who has ulcerative colitis and is scheduled for
surgery to create an ileostomy. After speaking with the provider the client says,” how will I
ever be able to have a normal life after this”? Which of the following responses should the
nurse make?
● "Everybody worries about how they will manage their ileostomy at first."
● "Are you worried that it will affect your relationship with your partner?"
● "Tell me how you think having the ileostomy will affect you."
● "This will cure your disease so you don't spend so much time in the hospital.”
Answer: "Tell me how you think having the ileostomy will affect you."
Rationale:
This response encourages the client to express their concerns and feelings about the
upcoming surgery and the potential impact of having an ileostomy on their life. It
demonstrates active listening and empathy, allowing the nurse to gain insight into the client's
perspective and address their specific worries and anxieties. By exploring the client's
thoughts and feelings, the nurse can provide personalized support and education to help the
client cope with the surgical procedure and its implications effectively.
71. A nurse is contributing to the plan of care for a client who has obsessive-compulsive
disorder regarding brushing his teeth. The client brushes his tongue several times a day and
has developed several ulcerations. Which of the following interventions should the nurse
identify as a priority?
● Assist the client in identifying his anxiety level.
● Speak to the client in a calm and soothing manner.
● Assist the client to identify triggers to obsessive behaviors.
● Provide information on stress reduction methods.
Answer: Assist the client in identifying his anxiety level.
Rationale:
Identifying the client's anxiety level is essential because obsessive-compulsive disorder
(OCD) is often driven by anxiety. By helping the client recognize when anxiety is heightened,
the nurse can implement appropriate coping strategies and interventions to manage anxiety
and reduce compulsive behaviors. Addressing anxiety is fundamental to managing OCD
symptoms effectively.

72. A nurse is collecting data from a client who is to begin taking alprazolam. Which of the
following findings should the nurse identify as a contradiction of this medication?
● Alcohol use disorder
● Drug withdrawal
● Seizure disorder
● Suicide attempts
Answer: Alcohol use disorder
Rationale:
Alprazolam, a benzodiazepine, should not be prescribed to individuals with alcohol use
disorder due to the increased risk of respiratory depression and central nervous system
depression when combined with alcohol. Combining alprazolam with alcohol can potentiate
the sedative effects of both substances, leading to serious adverse reactions, including
respiratory depression, coma, and death. Therefore, it is contraindicated in individuals with
alcohol use disorder.
73. A nurse is caring for a client who was admitted with delirium five days ago. The client
seeks permission from the nurse before performing ADLs. Which of the following actions
should the nurse take?
● Prepare the client for discharge.
● Quiz the client with orientation questions.
● Allow the client to function independently.
● Determine the client's level of awareness.
Answer: Determine the client's level of awareness.
Rationale:
Seeking permission before performing activities of daily living (ADLs) may indicate a
decreased level of awareness or confusion associated with delirium. The nurse should assess
the client's cognition and level of consciousness to determine their ability to make decisions
and perform ADLs safely. Understanding the client's level of awareness helps guide care
planning and interventions to promote safety and optimal functioning.
74. A nurse is caring for a client who has substance use disorder and undergoing
detoxification for alcohol use. Which of the following medications should the nurse use to
minimize the client's craving for alcohol?
● Chlordiazepoxide
● Atenolol
● Clonidine

● Carbamazepine
Answer: Chlordiazepoxide
Rationale:
Chlordiazepoxide is a benzodiazepine medication commonly used during alcohol
detoxification to manage withdrawal symptoms and minimize cravings for alcohol. It acts by
binding to gamma-aminobutyric acid (GABA) receptors, exerting anxiolytic and sedative
effects that help alleviate withdrawal symptoms such as anxiety, agitation, and autonomic
hyperactivity. By reducing cravings and withdrawal discomfort, chlordiazepoxide supports
the client through the detoxification process and promotes treatment adherence.
75. A nurse is contributing to the plan of care for an adolescent who is admitted for anorexia
nervosa and is at 50% of her ideal body weight. Which of the following interventions should
the nurse include in the plan?
● Encourage weekly weight gain of 6 lb. for the first 2 weeks.
● Set a specific duration for mealtimes.
● Avoid giving the client liquid supplements.
● Allow the client privacy during meals in order to promote trust.
Answer: Set a specific duration for mealtimes.
Rationale:
Setting a specific duration for mealtimes helps establish structure and consistency in the
client's eating routine, promoting regularity and reducing the likelihood of mealtime conflicts
or avoidance behaviors. By providing clear expectations regarding meal duration, the nurse
encourages the client to engage in the eating process and complete meals within a reasonable
timeframe. Structured mealtimes also support the development of healthy eating habits and
reinforce the importance of regular nutrition intake in the treatment of anorexia nervosa.
76. A nurse In an acute care facility is caring for an older adult client who is being discharged
to an extended care nursing facility the next day. The client asked,” why do I have to go to
that place”? Which of the following responses to the nurse make?
● “Your doctor feels that this is the best place for you right now.”
● “Why don't you ask your doctor about that when he comes in to see you?”
● “What have you been told about going to the new facility?”
● “Your family can't take care of you at home, so you will need to go there.”
Answer: “What have you been told about going to the new facility?”
Rationale:

This response promotes client-centered communication and encourages the client to express
their concerns and understanding about the transition to an extended care nursing facility. By
eliciting the client's perspective, the nurse can address any misconceptions, provide
clarification, and offer support tailored to the client's needs and preferences. Open-ended
questions facilitate therapeutic communication and empower the client to actively participate
in decision-making regarding their care and transition process.
77. A nurse is caring for a client who has dementia. When performing a mental status
examination (MSE), the nurse should include which of the following? SATA
● Ability to perform calculations
● Level of consciousness
● Presence of suicidal thoughts
● Long-term memory
● Level of orientation
Answer: Level of consciousness, Long-term memory, Level of orientation
Rationale:
In assessing the mental status of a client with dementia, it is important to evaluate various
cognitive functions and indicators of mental functioning. Including components such as level
of consciousness, long-term memory, and level of orientation provides valuable information
about the client's cognitive abilities, orientation to time, place, and person, and overall mental
status. While the presence of suicidal thoughts may be relevant in certain contexts, it is not
typically a component of a standard mental status examination for clients with dementia.
78. A nurse is assisting with the development of an in-service about conversion disorder and
adolescents. Which of the following information should the nurse include in the teaching?
● "Conversion disorder is under the adolescent's voluntary control."
● "Conversion disorder symptoms are irreversible."
● "Conversion disorder is characterized by an extreme fear of a serious illness."
● "Conversion disorder can be caused by posttraumatic stress disorder."
Answer: "Conversion disorder can be caused by posttraumatic stress disorder."
Rationale:
Conversion disorder is a somatic symptom disorder characterized by neurological symptoms
that cannot be attributed to a medical condition or substance use and are not explained by
neurological or medical disorders. It often manifests as motor or sensory disturbances that are
inconsistent with neurological patterns and are not under voluntary control. While the
etiology of conversion disorder is multifactorial and may involve psychological factors such

as stress and trauma, it is not considered to be under voluntary control, and symptoms are
typically reversible with appropriate interventions.
79. A nurse in an acute care mental health facility is caring for a client who is experiencing an
acute manic episode. What is the nurse's priority intervention?
● Discourage the client's inappropriate sexual expression.
● Control the client's use of loud and vulgar language.
● Maintain the client's contact with family members.
● Protect the client and others from impulsive behavior.
Answer: Protect the client and others from impulsive behavior.
Rationale:
During an acute manic episode, clients may exhibit impulsive and risky behaviors that can
endanger themselves or others. Therefore, the nurse's priority intervention is to ensure the
safety of the client and those around them by preventing impulsive actions such as reckless
spending, substance abuse, or self-harm. Implementing measures to protect the client from
harm, such as providing close supervision, removing potentially dangerous objects, and
establishing a safe environment, is essential in managing acute manic symptoms and
preventing adverse outcomes.
80. A nurse is caring for four clients in an acute care mental health facility. The nurse should
recognize which of the following clients as a potential candidate for electroconvulsive
therapy (ECT) as an effective treatment option?
● A client who has severe opioid use disorder
● A client who has bipolar disorder with rapid cycling
● A client who has long-standing antisocial personality disorder
● A client who has anorexia nervosa with electrolyte imbalances
Answer: A client who has bipolar disorder with rapid cycling
Rationale:
Electroconvulsive therapy (ECT) is considered an effective treatment option for certain
psychiatric conditions, particularly when other treatments have been ineffective or when
rapid symptom relief is needed. It is commonly used in severe cases of mood disorders such
as bipolar disorder, especially when there is rapid cycling or acute suicidal ideation. ECT has
been shown to be beneficial in stabilizing mood, reducing symptoms of depression or mania,
and improving overall functioning in clients with bipolar disorder. It is typically not indicated
for personality disorders or substance use disorders.

81. A nurse is caring for a client who has received nine electroconvulsive therapy treatments
and reports short-term memory loss. Which of the following responses to the nurse is
appropriate?
● "Memory loss is common and usually improves after a few weeks."
● "You should focus on how much better you feel."
● "I am going to notify your provider about your memory loss."
● "You will likely experience long-term memory loss as well."
Answer: "I am going to notify your provider about your memory loss."
Rationale:
It is essential for the nurse to notify the provider about the client's report of short-term
memory loss after receiving multiple electroconvulsive therapy (ECT) treatments. While
some degree of memory loss is expected following ECT, especially immediately after
treatment, persistent or worsening memory impairment may indicate a need for adjustments
to the treatment regimen or further evaluation. Notifying the provider allows for appropriate
assessment and intervention to address the client's concerns and ensure the safety and
effectiveness of ECT.
82. A nurse is assisting with the admission of a client who has bipolar disorder and is
experiencing an acute depressive episode. Which of the following prescriptions should the
nurse anticipate?
● Amitriptyline
● Lithium carbonate
● Phenelzine
● Mirtazapine
Answer: Mirtazapine
Rationale:
Mirtazapine is an antidepressant medication commonly used to treat depression, including
acute depressive episodes associated with bipolar disorder. It works by increasing the levels
of serotonin and norepinephrine in the brain, which helps improve mood and alleviate
depressive symptoms. Unlike lithium carbonate, which is primarily used for bipolar disorder's
manic phase, mirtazapine is indicated for depressive episodes. Phenelzine is a monoamine
oxidase inhibitor (MAOI) antidepressant, and amitriptyline is a tricyclic antidepressant, but
these medications are less commonly used as first-line treatments for bipolar depression due
to their side effect profiles and safety concerns.

83. A nurse is contributing to the plan of care for a client with paranoid schizophrenia. Which
of the following interventions should the nurse recommend to be included in the plan of care?
● Rotate the client's staff assignments daily.
● Use touch to calm the client during periods of anxiety.
● Serve food family-style rather than on individual plates.
● Assume an upbeat, friendly attitude when talking with clients.
Answer: Rotate the client's staff assignments daily.
Rationale:
Rotating the client's staff assignments daily helps minimize the development of paranoid
delusions related to staff members. Consistency in caregiving is essential for establishing
trust and rapport with clients; however, in paranoid schizophrenia, a change in routine or
personnel can reduce the risk of the client developing fixed false beliefs about specific staff
members. Rotating assignments promotes a therapeutic environment and reduces the
likelihood of paranoid ideation.
84. A nurse in an urgent care clinic is caring for a client who is using loud and rapid speech
and continuously repeats,” I don’t know why my wife left me”. Which of the following levels
of anxiety is the client experiencing?
● Mild
● Moderate
● Severe
● Panic
Answer: Severe (Characteristics include confusion, feelings of impending doom,
hyperventilation, tachycardia, withdrawal, loud and rapid speech, and aimless activity)
Rationale:
The client's symptoms, including loud and rapid speech, repetitive statements, and a
preoccupation with personal distress, suggest severe anxiety. Severe anxiety is characterized
by pronounced physiological and psychological manifestations, such as confusion, feelings of
impending doom, hyperventilation, tachycardia, withdrawal, and aimless activity. The client's
inability to focus or regulate their behavior reflects the severity of their anxiety and the need
for immediate intervention to alleviate distress and restore functioning.
85. A nurse is talking to a client following a cesarean delivery due to a failure to progress in
labor. The client expressed her disappointment that she was unable to have a natural
childbirth. Which of the following responses to the nurse is appropriate?
● “It sounds like you are feeling sad that things didn't go as planned."

● "At least you know you have a healthy baby."
● "Maybe next time you can have a vaginal delivery."
● "You will be able to resume sex sooner than if you had delivered vaginally."
Answer: “It sounds like you are feeling sad that things didn't go as planned."
Rationale:
This response acknowledges the client's feelings of disappointment and validates her
emotional experience without minimizing or dismissing her concerns. It demonstrates
empathy and active listening, which are essential components of therapeutic communication.
By acknowledging the client's emotions, the nurse can establish rapport, foster trust, and
provide support as the client processes her childbirth experience and adjusts to the outcome.
86. A nurse is teaching a newly licensed nurse about electroconvulsive therapy. Which of the
following statements by the newly licensed nurse indicates understanding?
● “ECT is an effective treatment for personality disorders.”
● “I should monitor the client closely for hypotension following ECT.”
● “Informed consent should be obtained prior to ECT.”
● “It is a myth that clients experience seizures during ECT.”
Answer: “Informed consent should be obtained prior to ECT.”
Rationale:
Obtaining informed consent is a crucial aspect of providing electroconvulsive therapy (ECT)
and ensures that clients understand the procedure, its risks, benefits, and alternatives before
giving their permission for treatment. Informed consent also involves discussing potential
side effects, such as memory loss, and addressing any questions or concerns the client may
have. Obtaining informed consent promotes autonomy, respects the client's right to make
decisions about their treatment, and enhances trust between the client and healthcare provider.
87. A nurse is talking with a client who has schizophrenia. Suddenly the client states,” I’m
frightened. Do you hear that? The voices are telling me to do terrible things”. Which of the
following responses by the nurse is appropriate?
● “You need to tell the voices to leave you alone.”
● “You need to understand that there are no voices.”
● “What are the voices telling you to do?”
● “Why do you think you are hearing the voices?”
Answer: “What are the voices telling you to do?”
Rationale:

This response demonstrates therapeutic communication by focusing on the client's subjective
experience and encouraging further exploration of the auditory hallucinations. By asking the
client about the content of the voices, the nurse gains insight into the nature and severity of
the hallucinations, which can inform the client's treatment plan. It also validates the client's
experience without challenging the reality of their perceptions or dismissing their concerns,
fostering trust and rapport in the therapeutic relationship.
88. A nurse is reinforcing teaching with a client who has a new prescription for amitriptyline.
Which of the following statements should the nurse include in the teaching?
● "You should change positions slowly while taking this medication."
● "This medication is prescribed to help overcome alcohol use disorder."
● "You should expect an increase in urine output."
● "This medication may cause increased salivation."
Answer: "You should change positions slowly while taking this medication."
Rationale:
Amitriptyline is a tricyclic antidepressant that can cause orthostatic hypotension, which is
characterized by a drop in blood pressure upon changing positions (e.g., from lying down to
standing up). Instructing the client to change positions slowly can help prevent dizziness,
light headed ness, or falls associated with orthostatic hypotension. This precaution is
important for client safety and minimizes the risk of injury while taking amitriptyline. The
other options are not associated with the side effects of amitriptyline.
89. A nurse in an acute mental health unit is assisting with the admission of a client who has
bipolar disorder. Which of the following findings indicates the client is experiencing acute
mania?
● The client's partner reports that the client has recently gained weight.
● The client is dressed in all black.
● The client responds to questions with disorganized speech.
● The client reports that voices are telling him to write a novel.
Answer: The client reports that voices are telling him to write a novel.
Rationale:
Acute mania is characterized by elevated, expansive, or irritable mood, along with increased
energy, impulsivity, and impaired judgment. Delusions and hallucinations, such as auditory
hallucinations commanding the individual to engage in grandiose or risky behaviors (e.g.,
writing a novel), are common manifestations of acute manic episodes in bipolar disorder. The
other options describe behaviors or symptoms that are not specific to acute mania.

90. A nurse is caring for a nine-year-old child who has a new diagnosis of diabetes Mellitus
and tells the nurse that he is eager to return to school and participate in sports. The mother
tells the nurse that she is afraid to let her take part in physical activities at school. Which of
the following responses should the nurse make?
● "Tell me more about how you are feeling about your daughter's activities."
● "You might want to consider teaching your daughter at home."
● "I agree. Her well-being is the most important thing."
● "You sound overprotective. Let's talk about this some more."
Answer: "Tell me more about how you are feeling about your daughter's activities."
Rationale:
This response acknowledges the mother's concerns and encourages further exploration of her
feelings and reasons for being afraid to allow her daughter to participate in physical activities
at school. By inviting the mother to share her perspective, the nurse can gain insight into her
specific fears or apprehensions, which may be related to her child's health condition or other
factors. Open communication and active listening promote collaboration between the nurse
and the mother in addressing the child's needs and ensuring a safe and supportive
environment for her participation in school activities.
91. A nurse is caring for a client who has invasive breast cancer and is starting chemotherapy.
She tells the nurse that she is worried about the adverse effects of the treatment. Which of the
following responses should the nurse make?
● "I will have your doctor discuss the adverse effects with you."
● "Don't worry. You will be feeling better in no time."
● "What is it about the adverse effects that concerns you?"
● "I agree. Sometimes the adverse effects can be worse than the disease."
Answer: "What is it about the adverse effects that concerns you?"
Rationale:
This response encourages the client to express her specific concerns and allows the nurse to
address them directly. It demonstrates active listening and shows empathy toward the client's
apprehensions, fostering trust and open communication. Understanding the client's worries
enables the nurse to provide tailored education and support to alleviate her anxieties about
chemotherapy.
92. A nurse is reinforcing teaching with a client who is scheduled to receive electroconvulsive
therapy. Which of the following statements should the nurse include in the teaching?
● "You might feel a bit confused and disoriented when you first wake up."

● "You may experience muscle cramping from the induced seizure."
● "You should expect to have ECT once per week for 6 weeks."
● "The most common adverse effects of ECT are related to the anesthesia."
Answer: "You might feel a bit confused and disoriented when you first wake up."
Rationale:
Confusion and disorientation are common transient side effects of electroconvulsive therapy
(ECT) immediately following the procedure. By informing the client about this potential
effect, the nurse prepares them for the experience and helps alleviate anxiety or distress
associated with waking up disoriented. Providing anticipatory guidance enhances the client's
understanding of the treatment process and promotes cooperation with post-ECT care
instructions.
93. A nurse is discussing group therapy with a client who has a depressive disorder. Which of
the following benefits of group therapy should the nurse include? SATA
● Focusing on the management of symptoms
● Decreasing feelings of isolation
● Gaining support and encouragement from other group members
● Developing social skills
● Assisting the client in choosing an antidepressant
Answer: Decreasing feelings of isolation, Gaining support and encouragement from other
group members, Developing social skills
Rationale:
Group therapy offers several benefits for clients with depressive disorders, including reducing
feelings of isolation by providing a supportive environment where individuals can share
experiences, emotions, and coping strategies with peers facing similar challenges. Group
members offer mutual support, encouragement, and validation, fostering a sense of belonging
and community. Additionally, group therapy provides opportunities for social interaction and
skill-building, such as practicing communication, problem-solving, and assertiveness skills in
a supportive context. While group therapy may indirectly support medication management by
providing emotional support and psychoeducation, it typically does not involve selecting
specific medications.
94. A nurse is caring for a school-age child who has a terminal illness. His parents told the
nurse they have reluctantly taken the child’s name off the list for participating in baseball this
year. Which of the following responses to the nurse make?
● "It must be frustrating for you to have to cancel an activity your son enjoyed."

● "Baseball can be a dangerous sport for children anyway."
● "You never know. He could be ready for baseball by next year."
● "Why did you feel like you needed to do that?"
Answer: "It must be frustrating for you to have to cancel an activity your son enjoyed."
Rationale:
This response acknowledges the parents' feelings of disappointment and empathizes with the
difficult decision they have made. It validates their emotions and demonstrates understanding
of their perspective, which can help strengthen the therapeutic relationship. Expressing
empathy and support fosters trust and open communication, enabling the nurse to provide
appropriate emotional support and assist the family in coping with their child's illness.
95. A nurse in an acute care mental health facility is caring for a hospitalized client who has
agoraphobia. The nurse observes the client is making progress when he is able to participate
in which of the following activities?
● Recreational therapy in the day room
● Daily group therapy sessions
● A picnic in a local park
● Lunch in the hospital cafeteria with family
Answer: A picnic in a local park
Rationale:
Agoraphobia involves fear or avoidance of situations or places where escape might be
difficult or help might not be available in the event of a panic attack or panic-like symptoms.
Progress in treating agoraphobia typically involves exposure therapy, gradually exposing the
client to feared situations or environments while providing support and coping strategies.
Participating in a picnic in a local park represents progress as it involves being outside of the
hospital environment and engaging in a social activity in a public space, which may have
been previously avoided due to agoraphobia.
96. A nurse in a mental health clinic is attempting to develop a therapeutic relationship with a
client. Which of the following actions should the nurse take?
● Set limits for the relationship.
● Promote the use of transference by the client.
● Instruct the client on how he should behave.
● Engage in friendly interactions with the client
Answer: Engage in friendly interactions with the client
Rationale:

Engaging in friendly interactions with the client fosters the development of a therapeutic
relationship built on trust, empathy, and mutual respect. Building rapport through friendly
interactions creates a safe and supportive environment where the client feels understood,
valued, and accepted. Setting limits for the relationship, promoting transference, or
instructing the client on how to behave may impede the therapeutic relationship by
undermining the client's autonomy, inhibiting open communication, or creating power
imbalances.
97. A nurse is caring for a hospitalized client who has bipolar disorder and is disturbing other
clients with incessant talking. Which of the following actions should the nurse take?
● Allow the client to interact freely with others on the unit.
● Assist the client to practice social interaction with peers during a community meeting.
● Escort the client to her room when she is observed trying to interact with other clients.
● Inform the client that restraints may be necessary if she cannot control her behavior.
Answer: Escort the client to her room when she is observed trying to interact with other
clients.
Rationale:
Incessant talking may disrupt the therapeutic milieu and compromise the safety and wellbeing of other clients on the unit. Escorting the client to her room provides a brief period of
separation from others, allowing her to regain composure and reduce stimulation. It also
prevents potential conflicts or disturbances while preserving the therapeutic environment for
all clients. Restraints should only be used as a last resort when less restrictive interventions
have been ineffective or when there is an imminent risk of harm to self or others.
98. A nurse is caring for a client who has schizophrenia. The client states, they lie about me
all the time and try to poison my food”. Which of the following responses to the nurse make?
● "Why do you think you are being lied about and poisoned?"
● "You are mistaken. Nobody is lying about you or trying to poison you."
● "You seem to be having some very frightening thoughts."
● "Who is lying about you and trying to poison you?"
Answer: "Who is lying about you and trying to poison you?"
Rationale:
This response acknowledges the client's concerns and invites further exploration of the
content of the delusions, which may provide insight into the client's perceptions and
experiences. By asking the client to identify the source of the perceived deception and
poisoning, the nurse demonstrates empathy, validates the client's reality, and encourages open

communication. It also helps to assess the severity and content of the delusions, which
informs the client's treatment plan and interventions.
99. A nurse is assisting with planning of care for a client following a suicide attempt. Which
of the following interventions is an appropriate suicide precaution?
● Remove utensils from the client's meal trays.
● Assign the client to a private room.
● Inspect the client's personal belongings.
● Tuck bed cover over the client's hands and arms.
Answer: Inspect the client's personal belongings.
Rationale:
Inspecting the client's personal belongings allows for the identification and removal of
potentially harmful items that could be used in future suicide attempts. This precaution helps
to create a safe environment and reduces the risk of self-harm. Removing utensils from meal
trays, assigning the client to a private room, and tucking bed covers over the client's hands
and arms are not specific suicide precautions and may not effectively address the underlying
risk factors for suicide.
100. The nurse is caring for a client who has been admitted involuntarily for psychiatric
treatment. Which of the following information about involuntary commitment should the
nurse provide the client's family?
● "A psychiatrist determines that the client's behavior is irrational."
● "The client has been accused of breaking the law."
● "The client's behavior is a threat to self or others."
● "The client is unable to manage the affairs necessary for daily life."
Answer: "The client's behavior is a threat to self or others."
Rationale:
Involuntary commitment typically occurs when a client's behavior poses a risk of harm to
themselves or others, and a mental health professional determines that involuntary
hospitalization is necessary to ensure the client's safety and well-being. Providing this
information to the client's family helps them understand the rationale behind the involuntary
commitment and emphasizes the importance of addressing the client's mental health needs in
a structured and supportive environment. The other options do not accurately reflect the
criteria for involuntary commitment and may lead to misunderstandings or misconceptions
about the legal and ethical aspects of psychiatric treatment.

101. A nurse is contributing to the plan of care for a client who is newly admitted with severe
depression. Which of the following actions should be added to the plan of care?
● Encourage the client to sleep during the day to make up for insomnia.
● Offer the client frequent small snacks during waking hours.
● Monitor the client for bouts of diarrhea.
● Schedule a brisk physical activity before bedtime to facilitate sleep.
Answer: Offer the client frequent small snacks during waking hours.
Rationale:
In severe depression, appetite disturbances are common, and some individuals may
experience decreased appetite leading to inadequate nutritional intake. Offering frequent
small snacks during waking hours can help ensure the client receives adequate nutrition
despite decreased appetite. This intervention supports the client's physical well-being and
contributes to overall health maintenance during the depressive episode. Encouraging
daytime sleep, monitoring for diarrhea, and scheduling physical activity before bedtime are
not appropriate interventions for managing severe depression.
102. A nurse in an outpatient clinic is caring for a client who reports bilateral knee pain after
a hike in the mountains this past weekend. He tells the nurse that he is worried because his
cousin died from bone cancer recently. Which of the following responses should the nurse
make?
● "I wouldn't worry about that. It's unlikely that you have cancer."
● "Why do you think your pain isn't just a result of the hike?"
● "I completely understand why you're concerned about this."
● "You seem worried. Let's talk about how you are feeling."
Answer: "You seem worried. Let's talk about how you are feeling."
Rationale:
This response acknowledges the client's feelings of worry and opens the door for further
exploration of his concerns. It demonstrates empathy and encourages the client to express his
feelings and thoughts about the situation, facilitating therapeutic communication. Dismissing
the client's concerns or making assumptions about his condition may invalidate his feelings
and hinder effective communication and trust-building.
103. A nurse is caring for a client who is experiencing delusions. Which of the following
actions should the nurse take?
● Interact with the client as if the content of the delusion were true.
● Acknowledge the feelings the client is experiencing regarding the delusion.

● Explain to the client that the delusional material is not possible.
● Explore the meaning of the delusion with the client.
Answer: Acknowledge the feelings the client is experiencing regarding the delusion.
Rationale:
Acknowledging the client's feelings regarding the delusion validates their emotional
experience without necessarily validating the content of the delusion itself. It demonstrates
empathy and fosters a therapeutic relationship while respecting the client's subjective reality.
Exploring the meaning of the delusion with the client may be appropriate in certain contexts,
but initially acknowledging their feelings is a more immediate and supportive intervention.
Interacting with the client as if the delusion were true may reinforce the delusional beliefs,
while explaining that the delusional material is not possible may lead to resistance or
defensiveness.
104. A nurse in an addiction rehabilitation center is contributing to the plan of care for a
newly admitted client who has alcohol use disorder. Which of the following interventions as a
nurse is priority?
● Pad the side rails of the bed with towels.
● Place the client in a private room.
● Accompany the client when ambulating.
● Determine the client's level of disorientation.
Answer: Accompany the client when ambulating.
Rationale:
Accompanying the client when ambulating is the priority intervention as it addresses safety
concerns related to the risk of falls or injury due to intoxication or withdrawal symptoms
associated with alcohol use disorder. Providing supervision during ambulation helps prevent
accidents and ensures the client's physical well-being. Padding the side rails, placing the
client in a private room, and determining the client's level of disorientation are important
aspects of care but are not immediate priorities in this situation.
105. A nurse is collecting data from a client who has schizophrenia. Which of the following
client findings should the nurse document as a positive symptom of the disorder?
● Dysphoria
● Anhedonia
● Disorganized speech
● Impaired judgment
Answer: Disorganized speech

Rationale:
Disorganized speech, such as incoherence, tangentiality, or derailment, is a positive symptom
of schizophrenia. Positive symptoms represent an excess or distortion of normal functions
and can include hallucinations, delusions, disorganized thinking/speech, and grossly
disorganized or catatonic behavior. Dysphoria and anhedonia are negative symptoms,
characterized by deficits in emotional expression and motivation. Impaired judgment is a
cognitive impairment and not specific to schizophrenia.
106. A nurse is assisting in the admission process for a client who has a history of violent
behavior. The client stands up and begins clenching her fist and arguing with the nurse.
Which of the following actions is the priority?
● Establish a therapeutic nurse-client relationship.
● Show the client around the unit and introduce her to other clients.
● Explore the truth of the client's statements.
● Set behavioral limits for the client.
Answer: Set behavioral limits for the client.
Rationale:
Setting behavioral limits for the client is the priority action to address the escalating situation
and ensure the safety of both the client and the staff. By establishing clear boundaries and
expectations for acceptable behavior, the nurse can prevent potential harm or violence. Once
the immediate safety concerns are addressed, the nurse can then work on establishing a
therapeutic nurse-client relationship and further assessing the client's needs. Showing the
client around the unit and exploring the truth of her statements may not be appropriate or
feasible during an agitated state.
107. A nurse is assessing a client in the emergency department who drank alcohol while
taking disulfiram. The client states, "the nurse told me not to drink when taking the
medication. I am just a social drinker. I didn’t realize I having just one drink with my friends
would cause such a problem". Which of the following defense mechanisms is a client
demonstrating?
● Denial
● Displacement
● Rationalization
● Reaction formation
Answer: Rationalization
Rationale:

Rationalization is a defense mechanism characterized by justifying or explaining one's
behavior or actions with logical reasoning to avoid facing the true motivations or
implications. In this scenario, the client is rationalizing their decision to drink alcohol while
taking disulfiram by minimizing the significance of the warning about potential adverse
reactions. Denial involves refusing to acknowledge or accept reality, displacement involves
redirecting emotions or impulses to a less threatening target, and reaction formation involves
expressing the opposite of one's true feelings or desires.
108. A nurse in an acute care mental health facility is caring for a client who has major
depression and states
, "everyone will be much better off when I’m gone". Which of the following actions should
the nurse take first?
● Assure that a staff member stays with the client at all times.
● Question the client about a suicide plan and method.
● Administer a prescribed anti-anxiety medication, instituting mouth checks to assure it is
swallowed.
● Inform the provider about the client's statement.
Answer: Question the client about a suicide plan and method.
Rationale:
Questioning the client about a suicide plan and method is the priority action to assess the
immediate risk of harm and determine the level of intervention required. This information
helps the nurse gauge the severity of the client's suicidal ideation and develop an appropriate
care plan to ensure the client's safety. While ensuring constant supervision and informing the
provider are important steps, directly addressing the client's statement and assessing for a
suicide plan take precedence to prevent potential self-harm.
109. A nurse caring for a client on an acute care mental health unit was involuntarily admitted
for 72 hours after attacking a neighbor. To keep the client in the hospital when the initial time
to hold the client expires, which of the following must be determined?
● Whether the client is a danger to herself or others.
● Whether the client is unwilling to accept that treatment is needed.
● Whether the client is unable to make arrangements to stay with someone.
● Whether the client is financially incapable of paying for prescribed medications.
Answer: Whether the client is a danger to herself or others.
Rationale:

To extend involuntary commitment beyond the initial hold period, it must be determined that
the client continues to pose a danger to themselves or others due to a mental illness. This
assessment is based on the client's current mental status and behavior, rather than their
willingness to accept treatment or financial resources. Involuntary commitment laws vary by
jurisdiction, but the primary consideration is ensuring the safety and well-being of the client
and others.
110. A nurse is collecting data from a group of clients who have anxiety disorders and have
prescriptions for various psychotropic medications. The nurse should recognize which of the
following clients as having an increased risk for suicide?
● A client who has an obsessive-compulsive disorder and takes fluoxetine.
● A client who has generalized anxiety disorder and takes diazepam.
● A client who has social anxiety disorder and takes propranolol.
● A client who has generalized anxiety disorder and takes diphenhydramine.
Answer: A client who has generalized anxiety disorder and takes diazepam.
Rationale:
Diazepam, a benzodiazepine, is associated with an increased risk of suicide due to its
potential to cause disinhibition, impulsivity, and paradoxical reactions, especially in clients
with underlying anxiety disorders. While other medications may also have side effects or
interactions that could contribute to suicidal ideation, benzodiazepines like diazepam are
specifically recognized for their association with increased suicide risk. Fluoxetine,
propranolol, and diphenhydramine are less likely to be associated with increased suicide risk
compared to benzodiazepines.
111. A nurse is assisting in the care of a client who has depressive disorder. Which of the
following interventions is the nurse's priority?
● Monitor for risk of self-harm.
● Administer prescribed antidepressants.
● Assist with activities of daily living.
Answer: Monitor for risk of self-harm.
Rationale:
In clients with depressive disorder, the risk of self-harm or suicide is a significant concern.
Monitoring for signs of suicidal ideation or behavior takes precedence to ensure the safety of
the client. Administering antidepressants and assisting with activities of daily living are
important aspects of care but are secondary to ensuring the client's safety.

112. A client who is depressed and has attempted suicide tells the nurse, “I should have died
because I am totally worthless.” Which of the following responses should the nurse make?
● "You've been feeling that your life has no meaning."
● "It's not unusual for people who have depression to feel this way."
● "Why do you feel you are worthless?"
● "You have a great deal to live for."
Answer: "It's not unusual for people who have depression to feel this way."
Rationale:
This response validates the client's feelings without judgment and acknowledges that such
feelings are common in individuals experiencing depression. It also opens the door for further
exploration and discussion about the client's emotions and experiences.
113. A nurse is caring for a client who has delusional behavior. The client states, “I can’t go
to group therapy today. I am expecting a high-level official to visit me!” Which of the
following responses should the nurse make?
● "Let's walk together to group so we can discuss your concerns."
● "Why do you think you are getting a visit from someone today?"
● "I wouldn't worry about that. They will call you back later."
● "I think you should attend group therapy. We will talk about this when you're done."
Answer: "Let's walk together to group so we can discuss your concerns."
Rationale:
This response acknowledges the client's belief while redirecting the focus to attending group
therapy. It offers support and encourages the client to participate in the scheduled activity
while addressing their concerns.
114. A nurse is caring for a client who is exhibiting signs of serotonin syndrome. Which of
the following is a nurse's priority intervention?
● Administering an anticonvulsant
● Administering diazepam
● Preparing for artificial ventilation
● Applying a cooling blanket
Answer: Applying a cooling blanket.
Rationale:
Serotonin syndrome is characterized by hyperthermia among other symptoms. Applying a
cooling blanket helps to reduce the client's body temperature and prevent further

complications associated with hyperthermia. While medications such as anticonvulsants and
diazepam may be used to manage symptoms, addressing hyperthermia is the priority.
115. A nurse is planning an interview for a newly admitted client and plans to include the
client's family members. Which of the following methods should the nurse use to determine
who to include in the interview?
● Include people who live in the same house with the client.
● Include people who are related to the client by blood and marriage.
● Include people whom the client views as family.
● Include people who can support the client adequately.
Answer: Include people whom the client views as family.
Rationale:
Involving individuals whom the client considers as family promotes a supportive and
therapeutic environment. This approach respects the client's perspective on familial
relationships and ensures that those present during the interview can provide meaningful
support and input.
116. A nurse caring for a client who is experiencing mania and is running around the unit.
Which of the following actions should the nurse take?
● Ask the client to put a puzzle together.
● Suggest the client exercise on a stationary bike.
● Take the client for a walk.
● Allow the client to run around to release extra energy.
Answer: Take the client for a walk.
Rationale:
Taking the client for a walk provides physical activity while also providing structure and
containment. Allowing the client to run around without direction may exacerbate the manic
behavior, while activities such as putting a puzzle together or exercising on a stationary bike
may not effectively redirect the excess energy.
117. A nurse is observing a newly licensed nurse as she assists in providing family therapy for
a client regarding relationship concerns with his spouse. Which of the following statements to
the client by the newly licensed nurse requires intervention by the nurse?
● “Tell me about the concerns that you have regarding your relationship.”
● “I think you should try to see your wife's point of view as well as your own.”
● “We should invite your partner to be a part of our discussion.”
● “Relationship difficulties are stressful and require effort to resolve.”

Answer: “I think you should try to see your wife's point of view as well as your own.”
Rationale:
While encouraging empathy and perspective-taking is beneficial, this statement may come
across as directive and potentially dismissive of the client's feelings. It's important to
facilitate exploration of the client's concerns rather than impose specific viewpoints.
118. A provider tells a client who has an anterior cruciate ligament that he may not play
football for the remainder of the season. The client yells at the provider, doesn't know what
he is talking about and kicks a chair. Which of the following defense mechanisms is the client
demonstrating?
● Denial
● Displacement
● Rationalization
● Reaction formation
Answer: Displacement
Rationale:
Displacement involves transferring emotions from their original source to a substitute target.
In this scenario, the client's anger and frustration about not being able to play football are
displaced onto the provider and the chair, which become the targets of his aggressive
behavior.
119. A nurse is caring for a client who has schizophrenia. Which of the following statements
by the client demonstrates concrete thinking?
● “I am aware that each problem has only one solution.”
● "I am a prophet of the most high judge."
● “The voices tell me that I must avoid large crowds.”
● “I know that you are trying to poison me and you can't convince me otherwise.”
Answer: “I am aware that each problem has only one solution.”
Rationale:
Concrete thinking is characterized by a literal interpretation of information without
abstraction or metaphorical understanding. The statement about each problem having only
one solution demonstrates a simplistic and rigid thought process, typical of concrete thinking.
120. A nurse is reviewing the nursing history for an adolescent client who has a new
diagnosis of conduct disorder. Which of the following findings is consistent with
characteristics of this disorder?
● Verbalizes presence of auditory hallucinations.

● Has frequent facial tics.
● Starts physical fights with peers at school.
● Develops signs of stress when school routines are changed.
Answer: Starts physical fights with peers at school.
Rationale:
Conduct disorder involves a persistent pattern of behavior that violates the basic rights of
others or major age-appropriate societal norms. Starting physical fights with peers at school is
consistent with the aggressive and antisocial behaviors seen in conduct disorder. Auditory
hallucinations, facial tics, and stress related to changes in school routines are not typically
associated with conduct disorder.
121. A nurse in a mental health facility is caring for a client who is readmitted for
schizophrenia after two weeks following the previous discharge. The nurse notices that the
client's appearance is unkempt and the client is mumbling to himself constantly. Which of the
following assessments as a nurse is priority?
● Mental status
● Anxiety level
● Perception of reality
● Physical health needs
Answer: Perception of reality
Rationale:
Given the client's diagnosis of schizophrenia and the observed symptoms of unkempt
appearance and constant mumbling, assessing the client's perception of reality is a priority.
This will help determine the severity of the psychotic symptoms and guide appropriate
interventions to address the client's distorted perceptions.
122. A nurse is talking with a newly licensed nurse about appropriate actions to take when a
client threatens to harm a specific individual. Which of the following statements by the newly
licensed nurse indicates understanding?
● "I need to make sure that the potential victim is notified."
● "I need to keep the information confidential due to the client's right to privacy."
● "I can only discuss the client's threats under court order."
● "I should place the client in seclusion."
Answer: "I need to make sure that the potential victim is notified."
Rationale:

When a client makes a specific threat to harm an individual, it is essential to ensure the safety
of the potential victim. Notifying the appropriate parties, such as the potential victim and
relevant healthcare team members, is necessary to implement safety measures and prevent
harm.
123. A nurse in an acute care mental health facility is caring for a client who commits suicide.
Which of the following is the priority intervention for staff following this incident?
● Attending a counseling session for staff members who provided care for the client.
● Encouraging other clients on the unit to talk about their feelings regarding the suicide.
● Identifying cues in the client's behavior that might have warned staff that he was
contemplating suicide.
● Recommending resources for the client's family to help them deal with their grief.
Answer: Identifying cues in the client's behavior that might have warned staff that he was
contemplating suicide.
Rationale:
Conducting a thorough assessment of the client's behavior leading up to the suicide is
essential for identifying potential warning signs or risk factors. This information can inform
improvements in care delivery and help prevent future suicides within the facility.
124. A nurse in an acute care mental health unit is speaking with the client who reports that
other clients leave trash in the lounge. Which of the following actions should the nurse take?
● Promise the client that the issue will be discussed at the next community meeting.
● Call housekeeping to clean up the lounge.
● Encourage the client to discuss the problem with other clients.
● Help the client clean up the dayroom.
Answer: Promise the client that the issue will be discussed at the next community meeting.
Rationale:
Promising to address the issue at the next community meeting involves the client in the
problem-solving process and provides an opportunity for collective discussion and resolution.
It also demonstrates responsiveness to the client's concerns while promoting a sense of
community responsibility.
125. A nurse is caring for a client who is scheduled to undergo abdominal surgery and tells
the nurse that he is very anxious about the operation. Which of the following actions should
the nurse take?
● Ask him to describe his concerns.
● Distract the client by giving him reading material.

● Suggest that he take a walk around the unit.
● Refer him to the spiritual care team.
Answer: Ask him to describe his concerns.
Rationale:
Encouraging the client to express his concerns allows the nurse to assess the specific sources
of anxiety related to the surgery. This facilitates open communication and enables the nurse to
address the client's fears and provide appropriate support and information.
126. A nurse is reviewing the medical record of a client who has a new prescription for
clozapine. Which of the following findings should the nurse identify as a contraindication for
this medication?
● Bone marrow depression
● Glaucoma
● Hypertension
● Urinary retention
Answer: Bone marrow depression
Rationale:
Clozapine can cause agranulocytosis, a serious condition characterized by a significant
decrease in white blood cell count. Therefore, it is contraindicated in clients with a history of
bone marrow depression or significant blood dyscrasias.
127. A nurse is discussing guided imagery with peers. Which of the following clients to the
nurse identify as being a candidate for guided imagery?
● A client who has post-traumatic stress disorder
● A client who has schizophrenia
● A client who has pedophilia
● A client who has paranoid personality disorder
Answer: A client who has post-traumatic stress disorder
Rationale:
Guided imagery is a relaxation technique that can help reduce stress and anxiety. It is often
used as part of therapy for clients with post-traumatic stress disorder (PTSD) to help them
cope with traumatic memories and associated symptoms.
128. A nurse who works in the community clinic facility is discussing suicide interventions
with a newly licensed nurse. Which of the following statements by the newly licensed nurse
indicates an understanding of a tertiary intervention?
● Nurses should perform screenings to identify clients at risk for suicide."

● "Nurses should recognize the lethality of a suicide plan."
● "Nurses should arrange counseling for the family following the suicide of a client."
● "Nurses should provide a safe environment to prevent the client from committing suicide."
Answer: "Nurses should arrange counseling for the family following the suicide of a client."
Rationale:
Tertiary interventions focus on providing support and interventions after a crisis has occurred.
Arranging counseling for the family following the suicide of a client is an example of a
tertiary intervention aimed at addressing the emotional and psychological needs of those
affected by the suicide.
129. A nurse is reinforcing teaching with a client who has depression about a new
prescription for fluoxetine. Which of the following statements by the client indicates
understanding of the teaching?
● "I should expect to feel better within 3 to 4 days."
● "I will increase my water intake up to 8 glasses a day."
● "I may experience sedation and sleepiness."
● "I will notice an improvement in my sex drive."
Answer: "I may experience sedation and sleepiness."
Rationale:
Sedation and sleepiness are common side effects of fluoxetine, a selective serotonin reuptake
inhibitor (SSRI). Understanding potential side effects is important for clients starting a new
medication regimen, as it allows them to anticipate and manage these effects appropriately.
130. A nurse is caring for a client who has paranoid delusions and believes the hospital food
is being poisoned by the staff. Which meal presentation should the nurse consider to be an
effective method of encouraging nutritional intake?
● Serve warm foods that arrive from the kitchen with lids in place.
● Serve the client's favorite foods in an attractive arrangement.
● Serve the same food that other clients in the dining room are eating.
● Serve individual items that have sealed packaging.
Answer: Serve individual items that have sealed packaging.
Rationale:
Given the client's paranoid delusions about the hospital food being poisoned, serving
individual items with sealed packaging can help alleviate concerns about contamination. This
approach provides a sense of safety and control over the food consumed, thereby encouraging
nutritional intake while respecting the client's beliefs.

131. A nurse on an acute care mental health unit is caring for a client who has generalized
anxiety disorder. The client receives an upsetting telephone call and is now rapidly pacing the
corridors of the unit. Which of the following actions should the nurse take?
● Walk with the client at a gradually slowing pace.
● Allow the client to pace alone until physically tired.
● Ask a small group of other clients to walk with the client.
● Calmly instruct the client to stop pacing and sit in the dayroom.
Answer: Walk with the client at a gradually slowing pace.
Rationale:
Walking with the client provides support and reassurance, while gradually slowing the pace
can help the client regulate their emotions and reduce anxiety. This approach acknowledges
the client's distress and offers a calming presence without escalating the situation.
132. A nurse is collecting data from a client whose child was killed two years ago. Which of
the following actions indicates that the client is experiencing maladaptive grieving?
● Visiting the child's grave every week
● Volunteering at a local children's hospital
● Talking about the child in the past tense
● Leaving the child's room exactly as it was before the loss.
Answer: Leaving the child's room exactly as it was before the loss.
Rationale:
Leaving the child's room unchanged may suggest that the client is unable to move forward in
the grieving process and is stuck in a state of denial or avoidance. This behavior can hinder
the client's ability to adapt to the loss and integrate it into their life.
133. A nurse overhears a client who has schizophrenia talking to herself. The client keeps
stating, "the muztracks are coming. The muztracks are coming." The nurse correctly
recognizes the client's use of the word "Muztrack" as an example of which of the following
alterations in speech?
● Clang association
● Neologism
● Word salad
Answer: Neologism
Rationale:

A neologism is a new word or expression created by the individual, often with a meaning that
is known only to them. In this case, "Muztrack" is a word created by the client, which does
not have a conventional meaning and is likely a product of their disordered thought process.
134. A nurse is caring for a client who has just been diagnosed with angina pectoris. The
client tells the nurse that he is afraid of dying from a heart attack. Which of the following is
an appropriate nursing response?
● "You will be prescribed medication that will keep you well for years to come."
● "Do you have family members who died from heart disease?"
● "Tell me more about these fears."
Answer: "Tell me more about these fears."
Rationale:
Encouraging the client to express their fears allows for open communication and provides an
opportunity for the nurse to address specific concerns and provide appropriate support and
education tailored to the client's individual needs.
135. A nurse is preparing to interview a client who has schizophrenia. Which of the following
actions should the nurse take?
● Sit on the other side of a table from the client.
● Place the client in a chair higher than the nurse.
● Start the interview with a question the client can answer with a "yes" or "no."
● Sit beside the client rather than facing him.
Answer: Sit beside the client rather than facing him.
Rationale:
Sitting beside the client fosters a sense of equality and partnership, which can help establish
rapport and facilitate communication. It also allows the nurse to observe the client's
nonverbal cues more effectively and promotes a comfortable environment for the interview.
136. A nurse is caring for a client with whom he has developed a therapeutic relationship, and
he will be discharged later in the day. The client thanks the nurse for his help during the
hospitalization. Which of the following responses should the nurse make?
● "Aren't you excited about being discharged today?"
● "How do you feel about being discharged?"
● "I will send you a note in a few weeks."
● "I know you will do well living out in the community."
Answer: "How do you feel about being discharged?"
Rationale:

This response encourages the client to express their feelings about the impending discharge,
which is essential for addressing any concerns or anxieties they may have and facilitating a
smooth transition to community living.
137. A nurse is reinforcing teaching with the client about manifestations of lithium toxicity.
Which of the following manifestations should the nurse include in the teaching?
● Vomiting and diarrhea
● Increased flatulence
● Loss of appetite
● Increased urination
Answer: Vomiting and diarrhea
Rationale:
Vomiting and diarrhea are common manifestations of lithium toxicity. Clients taking lithium
should be educated about the signs and symptoms of toxicity and advised to seek medical
attention if they experience these gastrointestinal symptoms.
138. A nurse in an acute care mental health facility is caring for a client who has depression
and has performed no ADLs since admission three days ago. The nurse observed that the
client is now wearing clean clothes and has combed her hair. Which of the following is an
appropriate response by the nurse?
● Oh, I'm so pleased that you finally put on clean clothes."
● "Why did you wear clean clothes and comb your hair today?"
● "That's good. You have on clean clothes and have combed your hair."
● "I see that you have on clean clothes and have combed your hair."
Answer: "That's good. You have on clean clothes and have combed your hair."
Rationale:
This response acknowledges the client's accomplishment in performing self-care activities,
which is important in depression management. It provides positive reinforcement without
imposing judgment or interrogation.
139. A nurse on an inpatient mental health unit is caring for a group of clients. Which of the
following actions by the nurse demonstrates the ethical concept of autonomy?
● Spending extra time to calm an agitated client
● Describing the adverse effects of a client's medications
● Supporting a client's wishes to refuse prescribed treatments
● Ensuring that a client understands expectations for group participation
Answer: Supporting a client's wishes to refuse prescribed treatments

Rationale:
Respecting a client's right to refuse treatment aligns with the ethical principle of autonomy,
which emphasizes an individual's right to make their own decisions about their healthcare. It
acknowledges the client's self-determination and promotes empowerment in the decisionmaking process.
140. A nurse is collecting data from an adolescent client who has anorexia nervosa. Which of
the following statements by the client is a sign of cognitive distortion?
● “I like to cut my food into small pieces.”
● “It's important for me to think about my strengths rather than my weaknesses.”
● “If I eat one piece of candy, I may as well eat ten.”
● “If I gain one more pound, I'll get to choose some menu items for my meals.”
Answer: “If I eat one piece of candy, I may as well eat ten.”
Rationale:
This statement reflects black-and-white thinking, a common cognitive distortion seen in
individuals with eating disorders such as anorexia nervosa. It involves viewing situations in
extremes, where one small action leads to an exaggerated and irrational consequence.
141. A nurse is collecting data from a client who has agoraphobia. Which of the following
prescriptions should the nurse anticipate the provider will prescribe?
● imipramine
● Haloperidol
● Verapamil
● Bromocriptine
Answer: imipramine
Rationale:
Imipramine is a tricyclic antidepressant commonly used in the treatment of agoraphobia and
other anxiety disorders. It helps alleviate symptoms such as panic attacks and avoidance
behavior associated with agoraphobia.
142. A nurse is collecting data from a newly admitted client who has schizophrenia. Which of
the following findings should the nurse identify as a negative manifestation of schizophrenia?
● Flat affect
● Lack of impulse control
● Circumstantial speech
● Somatic delusions
Answer: Flat affect

Rationale:
Flat affect refers to a lack of emotional expression, which is a negative symptom commonly
seen in schizophrenia. Negative symptoms involve a decrease or absence of normal functions.
143. A nurse is caring for a client who has an anxiety disorder and has begun to
hyperventilate, wring her hands, and is pacing the floor continually. Which of the following
actions should the nurse take first?
● Ask the client what precipitated this anxiety.
● Offer the client a prescribed anti-anxiety medication.
● Tell the client you will remain with her.
● Take the client to a quiet room.
Answer: Take the client to a quiet room.
Rationale:
Removing the client from a stimulating environment to a quiet room helps reduce sensory
input, which can help decrease anxiety. This action provides a calming environment for the
client to regain control.
144. A nurse is caring for an older adult client who has a prescription for lorazepam 0.5 mg.
Which of the following findings should the nurse report to the provider immediately?
● Increased anxiety (paradoxical effect)
● Anorexia
● Blurred vision
● Disorientation
Answer: Disorientation
Rationale:
Disorientation in an older adult can indicate an adverse reaction to lorazepam, potentially
indicating overdose or other serious issues. It requires immediate attention from the provider
to prevent further complications.
145. A nurse is caring for a client who has obsessive-compulsive disorder and is constantly
picking up after others and cleaning in the day room. The nurse should recognize the client's
actions as which of the following?
● Manipulating and controlling others' behavior.
● Focusing attention on useful tasks.
● Decreasing anxiety to a tolerable level.
● Limiting the amount of time available for interaction with others.
Answer: Decreasing anxiety to a tolerable level.

Rationale:
Engaging in cleaning and organizing behaviors is a common coping mechanism for
individuals with obsessive-compulsive disorder (OCD). These actions help reduce anxiety
and provide a sense of control over their environment.
146. A nurse is reinforcing dietary teaching with a client who has a new prescription for a
monoamine oxidase inhibitor. Which of the following foods should the nurse instruct the
client to avoid while taking an MAOI?
● Aged cheese
● Milk
● Shellfish
● Canned tuna
Answer: Aged cheese
Rationale:
Monoamine oxidase inhibitors (MAOIs) interact with tyramine-containing foods, such as
aged cheese, and can lead to a hypertensive crisis. Educating the client to avoid these foods is
essential to prevent serious complications.
147. A nurse is assisting with the admission of an older adult who is confused. Which of the
following statements by the client partner indicates that the client may be experiencing
delirium?
● "Her behavior changed so quickly, I wasn't sure what was happening."
● "She became very withdrawn and extremely sad."
● "Her speech was slow and repetitious."
● "She's been making up stories the past few weeks."
Answer: "Her behavior changed so quickly, I wasn't sure what was happening."
Rationale:
Delirium is characterized by acute onset confusion and changes in cognitive function. Rapid
changes in behavior or mental status are indicative of delirium rather than other cognitive
disorders.
148. A nurse is assisting with the admission of a client who has a suspected cognitive
disorder. Which of the following resources should be included as part of the data collection?
● Mini Mental State Examination (MMSE)
● The Scale for Assessment of Negative Symptoms (SANS)
● Abnormal Involuntary Movements Scale (AIMS)
● Hamilton Anxiety Scale

Answer: Mini Mental State Examination (MMSE)
Rationale:
The MMSE is a commonly used tool for assessing cognitive function. It evaluates
orientation, memory, attention, and other cognitive domains, aiding in the diagnosis and
assessment of cognitive disorders such as dementia.
149. A nurse is caring for a client three days after admission to an acute care mental health
facility for treatment of major depression. The client leaves her current activity, approaches
the nurse and states, "there’s no reason to go on living. I just want to end it all." Which of the
following nursing interventions is appropriate?
● Ask the client if she has a plan to commit suicide.
● Recognize the attempt at manipulation and escort the client back to her activity.
● Assist the client to her room and allow her to rest before resuming activity.
● Notify the client's family and request a visitor to stay with the client until thoughts of
suicide are gone.
Answer: Ask the client if she has a plan to commit suicide.
Rationale:
Assessing the presence of suicidal ideation and a plan is crucial for determining the level of
risk and initiating appropriate interventions. This question helps gauge the severity of the
client's suicidal thoughts and informs the nurse's next steps in ensuring the client's safety.

ATI MENTAL HEALTH PROCTORED

1. A charge nurse is discussing mental status exams with a newly licensed nurse. Which of
the following statements by the newly licensed nurse indicates an understanding of the
teaching? (Select all that apply)
A. "To assess cognitive ability, I should ask the client to count backward by sevens."
B. "To assess affect, I should observe the client's facial expression."
C. "To assess language ability, I should instruct the client to write a sentence."
D. "To assess remote memory, I should have the client repeat a list of objects."
E. "To assess the client's abstract thinking, I should ask the client to identify our most recent
presidents."
Answer: B, C, D
Rationale:

● Observing the client's facial expression is an appropriate method to assess affect, which
refers to the client's emotional expression.
● Instructing the client to write a sentence helps assess language ability, including syntax,
grammar, and coherence.
● Having the client repeat a list of objects assesses remote memory, which refers to the recall
of past events or information.
2. A nurse is planning care for a client who has a mental health disorder.
Which of the following actions should the nurse include as a psychobiological intervention?
A. Assist the client with systematic desensitization therapy.
B. Teach the client appropriate coping mechanisms.
C. Assess the client for comorbid health conditions.
D. Monitor the client for adverse effects of the medications.
Answer: D
Rationale:
● Monitoring the client for adverse effects of medications is a psychobiological intervention,
as it involves the biological aspect of mental health treatment, addressing the physiological
effects of medications on the client's body.
3. A nurse in an outpatient mental health clinic is preparing to conduct an initial client
interview. When conducting the interview, which of the following actions should the nurse
identify as the priority?
A. Coordinate holistic care with social services.
B. Identify the client's perception of her mental health status.
C. Include the client's family in the interview.
D. Teach the client about her current mental health disorder.
Answer: B
Rationale:
● Identifying the client's perception of her mental health status is the priority during the
initial interview, as it helps establish rapport, understand the client's perspective, and gather
essential information for treatment planning.
4. A nurse is told during change of shift report that a client is stuporous. When assessing the
client, which of the following findings should the nurse expect?
A. The client arouses briefly in response to a sternal rub.
B. The client has a Glasgow coma scale score less than 7.
C. The client exhibits decorticate rigidity.

D. The client is alert but disoriented to time and place.
Answer: A
Rationale:
● Stupor is a state of partial or nearly complete unconsciousness where the client can be
aroused briefly by vigorous stimuli such as a sternal rub.
5. A nurse is planning a peer group discussion about the DSM-5. Which of the following
information is appropriate to include in the discussion? (Select all that apply)
A. The DSM-5 includes client education handouts for mental health disorders.
B. The DSM-5 establishes diagnostic criteria for individual mental health disorders.
C. The DSM-5 indicates recommended pharmacological treatment for mental health
disorders.
D. The DSM-5 assists nurses in planning care for client's who have mental health disorders.
E. The DSM-5 indicates expected assessment findings of mental health disorders.
Answer: B, D, E
Rationale:
● The DSM-5 establishes diagnostic criteria for individual mental health disorders, guiding
clinicians in making diagnoses.
● The DSM-5 assists nurses in planning care for clients who have mental health disorders by
providing diagnostic criteria and guiding treatment decisions.
● The DSM-5 includes information on expected assessment findings of mental health
disorders, aiding in clinical assessment and diagnosis.
6. A nurse in an emergency mental health facility is caring for a group of clients.
The nurse should identify that which of the following clients requires a temporary emergency
admission?
A. A client who has schizophrenia with delusions of grandeur
B. A client who has manifestations of depression and attempted suicide a year ago
C. A client who has borderline personality disorder and assaulted a homeless man with a
metal rod
D. A client who has bipolar disorder and paces quickly around the room while talking to
himself
Answer: C
Rationale:

● A client who has borderline personality disorder and has recently assaulted a homeless man
with a metal rod poses an imminent danger to others, requiring temporary emergency
admission for safety and stabilization.
7. A nurse decides to put a client who has a psychotic disorder in seclusion overnight because
the unit is very short-staffed, and the client frequently fights with other clients. The nurse's
actions are an example of which of the following torts?
A. Invasion of privacy
B. False imprisonment
C. Assault
D. Battery
Answer: B
Rationale:
● Putting a client in seclusion without proper justification and against their will, as in this
scenario, constitutes false imprisonment, which is a legal tort.
8. A client tells a nurse, "Don't tell anyone but I hid a sharp knife under my mattress in order
to protect myself from my roommate, who is always yelling at me and threatening me."
Which of the following actions should the nurse take?
A. Keep the client's communication confidential, but talk to the client daily, using therapeutic
communication to convince him to admit to hiding the knife.
B. Keep the client's communication confidential, but watch the client and his roommate
closely.
C. Tell the client that this must be reported to the health care team because it concerns the
health and safety of the client and others.
D. Report the incident to the health care team, but do not inform the client of the intention to
do so.
Answer: C
Rationale:
● The nurse has a duty to report any threat of harm to the client or others, including the
presence of a weapon, to ensure the safety of everyone involved.
9. A nurse is caring for a client who is in mechanical restraints. Which of the following
statements should the nurse include in the documentation? (Select all that apply)
A. "Client ate most of his breakfast."
B. "Client was offered 8 oz of water every hr."
C. "Client shouted obscenities at assistive personnel."

D. "Client received chlorpromazine 15 mg by mouth at 1000."
E. "Client acted out after lunch."
Answer: C, D, E
Rationale:
● Documenting any aggressive or disruptive behavior, such as shouting obscenities, is
important for assessing the client's response to treatment and ensuring appropriate
interventions.
● Documenting medication administration, including type and dosage, is crucial for
maintaining accurate records of the client's treatment.
● Documenting any behavioral changes or incidents, such as acting out, helps track the
client's progress and response to interventions.
10. A nurse hears a newly licensed nurse discussing a client's hallucinations in the hallway
with another nurse. Which of the following actions should the nurse take first?
A. Notify the nurse manager.
B. Tell the nurse to stop discussing the behavior.
C. Provide an in-service program about confidentiality.
D. Complete an incident report.
Answer: B
Rationale:
● The immediate action should be to address the breach of confidentiality by instructing the
nurse to stop discussing the client's confidential information in a public area.
11. A nurse is caring for the parents of a child who has demonstrated changes in behavior and
mood. When the mother of the child asks the nurse for reassurance about her son's condition,
which of the following responses should the nurse make?
A. "I think your son is getting better. What have you noticed?"
B. "I'm sure everything will be okay. It just takes time to heal."
C. "I'm not sure what’s wrong. Have you asked the doctor about your concerns?"
D. "I understand you're concerned. Let's discuss what concerns you specifically."
Answer: D
Rationale:
● This response acknowledges the mother's concerns and invites further discussion to address
her specific worries, facilitating open communication and support.

12. A nurse is caring for a client who smokes and has lung cancer. The client reports, "I'm
coughing because I have that cold that everyone has been getting." The nurse should identify
that the client is using which of the following defense mechanisms?
A. Reaction formation
B. Denial
C. Displacement
D. Sublimation
Answer: B
Rationale:
● Denial involves refusing to acknowledge the existence of a real situation or the feelings
associated with it, such as denying the link between smoking and lung cancer.
13. A nurse is providing preoperative teaching for a client who was just informed that she
requires emergency surgery. The client has a respiratory rate 30/min and says, "This is
difficult to comprehend. I feel shaky and nervous."
The nurse should identify that the client is experiencing which of the following levels of
anxiety?
A. Mild
B. Moderate
C. Severe
D. Panic
Answer: C
Rationale:
● Severe anxiety is characterized by increased respiratory rate, difficulty concentrating,
trembling, and feeling nervous or shaky.
14. A nurse is caring for a client who is experiencing moderate anxiety.
Which of the following actions should the nurse take when trying to give necessary
information to the client? (Select all that apply.)
A. Reassure the client that everything will be okay.
B. Discuss prior use of coping mechanisms with the client.
C. Ignore the client's anxiety so that she will not be embarrassed.
D. Demonstrate a calm manner while using simple and clear directions.
E. Gather information from the client using closed-ended questions.
Answer: B, D
Rationale:

● Discussing prior use of coping mechanisms helps empower the client and provides
strategies to manage anxiety effectively.
● Demonstrating a calm manner and using simple, clear directions helps reduce the client's
anxiety and facilitates understanding.
15. A nurse is talking with a client who is at risk for suicide following the death of his spouse.
Which of the following statements should the nurse make?
A. "I feel very sorry for the loneliness you must be experiencing."
B. "Suicide is not the appropriate way to cope with loss."
C. "Losing someone close to you must be very upsetting."
D. "I know how difficult it is to lose a loved one."
Answer: C
Rationale:
● This statement acknowledges the client's feelings without judgment and validates the
emotional impact of losing someone close, fostering empathy and understanding.
16. A charge nurse is discussing the characteristics of a nurse-client relationship with a newly
licensed nurse. Which of the following characteristics should the nurse include in the
discussion? (Select all that apply)
A. The needs of both participants are met.
B. An emotional commitment exists between the participants.
C. It is goal-directed.
D. Behavioral change is encouraged.
E. A termination date is established.
Answer: A, C, D
Rationale:
● In a therapeutic relationship, the needs of both the nurse and the client should be addressed
to foster mutual understanding and growth.
● A therapeutic relationship is goal-directed, focusing on achieving specific outcomes related
to the client's well-being and recovery.
● Encouraging behavioral change is a key aspect of a therapeutic relationship, aiming to
facilitate positive changes in the client's thoughts, feelings, and actions.
17. A nurse is in the working phase of a therapeutic relationship with a client who has
methamphetamine use disorder. Which of the following actions indicates transference
behavior?
A. The client asks the nurse whether she will go out to dinner with him.

B. The client accuses the nurses of telling him what to do just like his ex-girlfriend.
C. The client reminds the nurse of a friend who died from a substance overdose.
D. The client becomes angry and threatens to harm himself.
Answer: B
Rationale:
● Transference occurs when the client unconsciously transfers feelings and attitudes from
significant others onto the nurse or therapist. Accusing the nurses of behaving like his exgirlfriend suggests transference.
18. A nurse is planning care for the termination phase of a nurse-client relationship. Which of
the following actions should the nurse include in the plan of care?
A. Discussing ways to use new behaviors
B. Practicing new problem-solving skills
C. Developing goals
D. Establishing boundaries
Answer: A
Rationale:
● During the termination phase, discussing ways to use new behaviors and skills learned
during therapy helps the client integrate these changes into their daily life.
19. A nurse is orienting a new client to a mental health unit. When explaining the unit's
community meetings, which of the following statements should the nurse make?
A. "You and a group of other clients will meet to discuss your treatment plans."
B. "Community meetings have a specific agenda that is established by staff."
C. "You and the other clients will meet with staff to discuss common problems."
D. "Community meetings are an excellent opportunity to explore your personal mental health
issues."
Answer: B
Rationale:
● Community meetings typically have a specific agenda established by staff and provide an
opportunity for clients to discuss common issues, learn coping strategies, and participate in
group therapy sessions.
20. A nurse is caring several clients who are attending community-based mental health
programs. Which of the following clients should the nurse plan to visit first?
A. A client who recently burned her arm while using a hot iron at home.

B. A client who requests that her antipsychotic medication be changed due to some new
adverse effects.
C. A client who says he is hearing a voice that tells him he is not worth living anymore.
D. A client who tells the nurse he experienced manifestations of severe anxiety before and
during a job interview.
Answer: C
Rationale:
● The client who is hearing voices telling him he is not worth living anymore is experiencing
potentially life-threatening symptoms and requires immediate intervention to assess for
suicidality and ensure safety.
21. A community mental health nurse is planning care to address the issue of depression
among older adult clients in the community. Which of the following interventions should the
nurse plan as a method of tertiary prevention?
A. Educating clients on health promotion techniques to reduce the risk of depression
B. Performing screenings for depression at community health programs
C. Establishing rehabilitation programs to decrease the effects of depression
D. Providing support groups for clients at risk for depression
Answer: C
Rationale:
● Tertiary prevention aims to reduce the impact of an already established disease or
condition. Establishing rehabilitation programs to decrease the effects of depression among
older adults in the community constitutes tertiary prevention by providing interventions to
manage and cope with existing depression.
22. A nurse is working in a community mental health facility. Which of the following services
does this type of program provide? (Select all that apply)
A. Educational groups
B. Medication dispensing programs
C. Individual counseling programs
D. Detoxification programs
E. Family therapy
Answer: A, C, E
Rationale:
• Educational groups are commonly offered in community mental health facilities to provide
information and support to individuals with mental health issues and their families.

• Individual counseling programs are essential in addressing the specific needs and concerns
of clients in a community mental health setting.
• Family therapy is often provided to address familial dynamics and support systems, which
play a crucial role in the recovery process of individuals with mental health issues.
23. A nurse in an acute mental health facility is assisting with discharge planning for a client
who has a severe mental illness and requires supervision much of the time. The client's wife
works all day but is home by late afternoon. Which of the following strategies should the
nurse suggest as appropriate follow-up care?
A. Receiving daily care from a home health aide
B. Having a weekly visit from a nurse case worker
C. Attending a partial hospitalization program
D. Visiting a community mental health center on a daily basis
Answer: A
Rationale:
• Given the client's need for supervision and the availability of the wife after late afternoon,
receiving daily care from a home health aide would ensure consistent support and supervision
for the client.
24. A nurse is caring for a group of clients. Which of the following clients should a nurse
consider for referral to an assertive community treatment (ACT) group?
A. A client in an acute care mental health facility who has fallen several times while running
down the hallway
B. A client who lives at home and keeps "forgetting" to come in for his monthly antipsychotic
injection for schizophrenia
C. A client in a day treatment program who says he is becoming more anxious during group
therapy
D. A client in a weekly grief support group who says she still misses her deceased husband
who has been dead for 3 months
Answer: B
Rationale:
• Assertive Community Treatment (ACT) is designed for individuals with severe mental
illness who require comprehensive, community-based support. The client who consistently
forgets to come in for their monthly antipsychotic injection would benefit from the intensive,
multidisciplinary approach of an ACT team to ensure medication compliance and support in
managing their illness in the community.

25. A nurse is teaching a client who has an anxiety disorder and is scheduled to begin
classical psychoanalysis. Which of the following client statements indicates an understanding
of this form of therapy?
A. "Even if my anxiety improves, I will need to continue this therapy for 6 weeks."
B. "The therapist will focus on my past relationships during our sessions."
C. "Psychoanalysis will help me reduce my anxiety by changing my behaviors."
D. "This therapy will address my conscious feelings about stressful experiences."
Answer: B
Rationale:
• Classical psychoanalysis typically involves exploring the client's unconscious mind and past
experiences, particularly focusing on early relationships and experiences. Option B reflects
an understanding of this aspect of psychoanalysis.
26. A nurse is discussing free association as a therapeutic tool with a client who has major
depressive disorder. Which of the following client statements indicates understanding of this
technique?
A. "I will write down my dreams as soon as I wake up."
B. "I may begin to associate my therapist with important people in my life."
C. "I can learn to express myself in a nonaggressive manner."
D. "I should say the first thing that comes to my mind."
Answer: D
Rationale:
• Free association involves the client saying whatever comes to their mind without censorship
or filtering. Option D reflects an understanding of this technique.
27. A nurse is preparing to implement cognitive reframing techniques for a client who has an
anxiety disorder. Which of the following techniques should the nurse include in the plan of
care? (Select all that apply)
A. Priority restructuring
B. Monitoring thoughts
C. Diaphragmatic breathing
D. Journal keeping
E. Meditation
Answer: B, D
Rationale:

• Monitoring thoughts involves identifying and challenging negative or irrational thoughts, a
key component of cognitive reframing.
• Journal keeping allows the client to track their thoughts and emotions, facilitating the
identification of patterns and cognitive distortions for further exploration and reframing.
28. A nurse is caring for a client who has a new prescription for disulfiram for treatment of
alcohol use disorder. The nurse informs the client that this medication can cause nausea and
vomiting if he drinks alcohol. Which of the following types of treatment is this method an
example?
A. Aversion therapy
B. Flooding
C. Biofeedback
D. Dialectical behavior therapy
Answer: A
Rationale:
• Disulfiram works by creating an aversion to alcohol consumption through the unpleasant
reaction (nausea and vomiting) that occurs when alcohol is ingested. This is a classic example
of aversion therapy.
29. A nurse is assisting with systematic desensitization for a client who has an extreme fear of
elevators. Which of the following actions should the nurse implement with this form of
therapy?
A. Demonstrate riding in an elevator, and then ask the client to imitate the behavior.
B. Advise the client to say "stop" out loud every time he begins to feel an anxiety response
related to an elevator.
C. Gradually expose the client to an elevator while practicing relaxation techniques.
D. Stay with the client in an elevator until his anxiety response diminishes.
Answer: C
Rationale:
• Systematic desensitization involves gradually exposing the client to the feared stimulus
(elevators in this case) while pairing it with relaxation techniques to reduce anxiety. Option C
describes this process accurately.
30. A nurse wants to use democratic leadership with a group whose purpose is to learn
appropriate conflict resolution techniques. The nurse is correct in implementing this form of
group leadership when she demonstrates which of the following actions?
A. Observes group techniques without interfering with the group process

B. Discusses a technique and then directs members to practice the technique
C. Asks for group suggestions of techniques and then supports discussion
D. Suggests techniques and asks group members to reflect on their use
Answer: C
Rationale:
• Democratic leadership involves facilitating group interaction and decision-making
processes. Option C demonstrates democratic leadership by allowing group members to
suggest techniques and supporting their discussion.
31. A nurse is planning group therapy for clients dealing with bereavement. Which of the
following activities should the nurse include in the initial phase? (Select all that apply)
A. Encourage the group to work toward goals
B. Define the purpose of the group
C. Discuss termination of the group
D. Identify informal roles of members within the group
E. Establish an expectation of confidentiality within the group
Answer: B, D, E
Rationale:
• Defining the purpose of the group helps establish direction and focus.
• Identifying informal roles within the group helps establish group dynamics and interactions.
• Establishing an expectation of confidentiality creates a safe environment for members to
share personal experiences and emotions.
32. A nurse working on an acute mental health unit forms a group to focus on selfmanagement of medications. At each of meetings, two of the members use the opportunity to
discuss their common interest in gambling on sports.
This is an example of which of the following concepts?
A. Triangulation
B. Group process
C. Subgroup
D. Hidden agenda
Answer: D
Rationale:
• This scenario describes a hidden agenda, where members divert the focus of the group away
from the intended topic towards their own interests without openly expressing their
intentions.

33. A nurse is conducting a family therapy session. The adolescent son tells the nurse that he
plans ways to make his sister look bad so his parents will think he's the better sibling, which
he believes will give him more privileges. The nurse should identify this dysfunctional
behavior as which of the following?
A. Placation
B. Manipulation
C. Blaming
D. Distraction
Answer: B
Rationale:
• Manipulation involves attempting to control or influence others' behaviors or decisions for
personal gain. In this scenario, the adolescent son's behavior of trying to make his sister look
bad to gain more privileges demonstrates manipulation.
34. A nurse is working with an established group and identifies various member roles. Which
of the following should the nurse identify as an individual role?
A. A member who praises input from other members
B. A member who follows the direction of other members
C. A member who brags about accomplishments
D. A member who evaluates the group's performance toward a standard
Answer: C
Rationale:
• Bragging about accomplishments is an individual role as it focuses on the member's own
achievements rather than facilitating group interaction or dynamics.
35. A nurse is preparing to provide an educational seminar on stress to other nursing staff.
Which of the following information should the nurse include in the discussion?
A. Excessive stressors cause the client to experience distress.
B. The body's initial adaptive response to stress is denial.
C. Absence of stressors results in homeostasis.
D. Negative, rather than positive, stressors produce a biological response.
Answer: A
Rationale:
• Excessive stressors can lead to distress, which is the negative psychological and emotional
response to stress. It's important for nursing staff to understand the distinction between stress
and distress to effectively manage stress in themselves and their patients.

36. A nurse is discussing acute vs prolonged stress with a client. Which of the following
effects should the nurse identify as an acute stress response? (Select all that apply)
A. Chronic pain
B. Depressed immune system
C. Increased blood pressure
D. Panic attacks
E. Unhappiness
Answer: C, D
Rationale:
• Acute stress responses are immediate and short-term reactions to stressors. Increased blood
pressure and panic attacks are typical acute stress responses characterized by physiological
arousal and heightened emotional reactions.
37. A nurse is teaching a client about stress-reduction techniques. Which of the following
client statements indicates understanding of the teaching?
A. "Cognitive reframing will help me change my irrational thoughts to something positive."
B. "Progressive muscle relaxation uses a mechanical device to help me gain control over my
pulse rate."
C. "Biofeedback causes my body to release endorphins so that I feel less stress and anxiety."
D. "Mindfulness allows me to prioritize the stressors that I have in my life so that I have less
anxiety."
Answer: A, D
Rationale:
• Cognitive reframing involves changing negative thought patterns to more positive and
adaptive ones, thus reducing stress.
• Mindfulness helps individuals focus on the present moment and gain perspective on
stressors, leading to reduced anxiety.
38. A client says she is experiencing increased stress because her significant other is
"pressuring me and my kids to go live with him. I love him, but I'm not ready to do that."
Which of the following recommendations should the nurse make to promote a change in the
client's situation?
A. Learn to practice mindfulness
B. Use assertiveness techniques
C. Exercise regularly
D. Rely on the support of a close friend

Answer: B
Rationale:
• Using assertiveness techniques can help the client communicate her needs and boundaries
effectively with her significant other, potentially resolving the source of stress in the
relationship.
39. A nurse is caring for a client who states, "I'm so stressed at work because of my coworker.
He expects me to finish his work because he's too lazy!" When discussing effective
communication, which of the following statements by the client to his coworker indicates
client understanding?
A. "You really should complete your own work. I don't think it's right to expect me to
complete your responsibilities."
B. "Why do you expect me to finish your work? You must realize that I have my own
responsibilities."
C. "It is not fair to expect me to complete your work. If you continue, then I will report your
behavior to our supervisor."
D. "When I have to pick up extra work, I feel very overwhelmed. I need to focus on my own
responsibilities."
Answer: C
Rationale:
• Option C communicates the client's assertiveness in expressing their boundaries and
consequences if the coworker continues to impose their responsibilities onto them.
40. A nurse is providing teaching for a client who is scheduled to receive ECT for the
treatment of major depressive disorder. Which of the following client statements indicates
understanding of the teaching?
A. "It is common to treat depression with ECT before trying medications."
B. "I can have my depression cured if I receive a series of ECT treatments."
C. "I should receive ECT once a week for 6 weeks."
D. "I will receive a muscle relaxant to protect me from injury during ECT."
Answer: D
Rationale:
• ECT (electroconvulsive therapy) is often accompanied by a muscle relaxant to prevent
injury during the induced seizures. Option D indicates the client's understanding of this safety
measure during the procedure.

41. A charge nurse is discussing TMS with a newly licensed nurse. Which of the following
statements by the newly licensed nurse indicates an understanding of the teaching?
A. "TMS is indicated for clients who have schizophrenia spectrum disorders."
B. "I will provide post-anesthesia care following TMS."
C. "TMS treatments usually last 5-10 minutes."
D. "I will schedule the client for daily TMS treatments for the first several weeks."
Answer: C
Rationale:
• Transcranial magnetic stimulation (TMS) treatments typically last between 5 to 10 minutes
per session. This duration is commonly observed in clinical practice.
42. A nurse is assessing a client immediately following an ECT procedure. Which of the
following findings should the nurse expect? (Select all that apply)
A. Hypotension
B. Paralytic ileus
C. Memory loss
D. Nausea
E. Confusion
Answer: C, D, E
Rationale:
• Memory loss, nausea, and confusion are common immediate post-procedure effects of
electroconvulsive therapy (ECT). These effects are typically transient and resolve shortly
after the procedure.
43. A nurse is leading a peer group discussion about the indications for ECT. Which of the
following indications should the nurse include in the discussion?
A. Borderline personality disorder
B. Acute withdrawal related to a substance use disorder
C. Bipolar disorder with rapid cycling
D. Dysphoric disorder
Answer: C
Rationale:
• Electroconvulsive therapy (ECT) is indicated for conditions such as severe depression,
bipolar disorder with rapid cycling, and some cases of schizophrenia, among others. Bipolar
disorder with rapid cycling is one of the accepted indications for ECT.

44. A nurse is planning care for a client following surgical implantation of a VNS device. The
nurse should plan to monitor for which of the following adverse effects? (Select all that
apply)
A. Voice changes
B. Seizure activity
C. Disorientation
D. Dysphagia
E. Neck pain
Answer: A, D, E
Rationale:
• Voice changes, dysphagia, and neck pain are potential adverse effects following surgical
implantation of a vagus nerve stimulation (VNS) device. Monitoring for these adverse effects
is essential for early detection and intervention.
45. A nurse observes a client who has OCD repeatedly applying, removing, and then
reapplying makeup. The nurse identifies that repetitive behavior in a client who has OCD is
due to which of the following underlying reasons?
A. Narcissistic behavior
B. Fear of rejection from staff
C. Attempt to reduce anxiety
D. Adverse effect of antidepressant medication
Answer: C
Rationale:
• Repetitive behaviors in OCD serve as coping mechanisms to alleviate anxiety or distress
associated with obsessions. The primary motivation behind such behaviors is to reduce
anxiety, rather than other factors like narcissism or fear of rejection.
46. A nurse is caring for a client who is experiencing a panic attack. Which of the following
actions should the nurse take?
A. Discuss new relaxation techniques
B. Show the client how to change his behavior
C. Distract the client with a television show
D. Stay with the client and remain quiet
Answer: D
Rationale:

• During a panic attack, the client may benefit from the presence of a calm and supportive
individual. The nurse should stay with the client, provide reassurance, and maintain a quiet
environment to help the client feel safe and supported until the panic attack subsides.
47. A nurse is assessing a client who has generalized anxiety disorder. Which of the following
findings should the nurse expect? (Select all that apply)
A. Excessive worry for 6 months
B. Impulsive decision making
C. Delayed reflexes
D. Restlessness
E. Need for reassurance
Answer: A, D, E
Rationale:
• Generalized anxiety disorder is characterized by excessive worry and anxiety about various
aspects of life for at least 6 months (option A).
• Restlessness and the need for reassurance are common manifestations of generalized
anxiety disorder due to persistent and uncontrollable anxiety (options D and E).
48. A nurse is caring for a client who has body dysmorphic disorder. Which of the following
actions should the nurse plan to take first?
A. Assessing the client's risk for self-harm
B. Instilling hope for positive outcomes
C. Encouraging the client to participate in group therapy sessions
D. Encouraging the client to participate in treatment decisions
Answer: A
Rationale:
• Assessing the client's risk for self-harm is the priority intervention to ensure the safety of the
client. Clients with body dysmorphic disorder may have distorted perceptions of their
appearance and may be at risk for self-harm or suicide due to distress related to their
perceived flaws.
49. A nurse is caring for a client who has acute stress disorder and is experiencing severe
anxiety. Which of the following statements should the nurse make?
A. "Tell me about how you are feeling right now."
B. "You should focus on the positive things in your life to decrease your anxiety."
C. "Why do you believe you are experiencing this anxiety?"
D. "Let's discuss the medications your provider is prescribing to decrease your anxiety."

Answer: A
Rationale:
• Encouraging the client to express their feelings provides an opportunity for the client to
ventilate emotions and helps the nurse gain insight into the client's current emotional state,
facilitating further assessment and intervention as needed.
50. A nurse working on an acute mental health unit is caring for a client who has PTSD.
Which of the following findings should the nurse expect? (Select all that apply)
A. Difficulty concentrating on tasks
B. Obsessive need to talk about the traumatic event
C. Negative self-image
D. Recurring nightmares
E. Diminished reflexes
Answer: A, B, C, D
Rationale:
• Clients with PTSD often experience difficulty concentrating (option A), an obsessive need
to talk about the traumatic event (option B), negative self-image (option C), and recurring
nightmares (option D) as part of their symptomatology. Diminished reflexes are not typically
associated with PTSD.
51. A nurse is involved in a serious and prolonged mass casualty incident in the emergency
department. Which of the following strategies should the nurse use to help prevent
developing a trauma-related disorder? (Select all that apply)
A. Avoid thinking about the incident when it is over
B. Take breaks during the incident for food and water
C. Debrief with others following the incident
D. Hold emotions in check in the days following the incident
E. Take advantage of offered counseling
Answer: B, C, E
Rationale:
• Taking breaks during the incident for food and water helps to maintain physical and
emotional well-being.
• Debriefing with others following the incident allows for processing of emotions and
experiences, which can help prevent the development of trauma-related disorders.
• Taking advantage of offered counseling provides an opportunity for additional support and
processing of emotions related to the incident.

52. A nurse is collecting an admission history for a client who has acute stress disorder
(ASD). Which of the following information should the nurse expect to collect?
A. The client remembers many details about the traumatic incident
B. The client expresses heightened elation about what is happening
C. The client states he first noticed manifestations of the disorder 6 weeks after the traumatic
incident occurred.
D. The client expresses a sense of unreality about the traumatic event
Answer: D
Rationale:
• Acute stress disorder (ASD) often involves a sense of unreality or detachment from oneself
or one's surroundings during or after the traumatic event, known as dissociation. This sense of
unreality is a characteristic symptom of ASD.
53. A nurse is caring for a client who has derealization disorder. Which of the following
findings should the nurse identify as an indication of derealization?
A. The client explains that her body seems to be floating above the ground
B. The client has the idea that someone is trying to kill her and steal her money
C. The client states that the furniture in the room seems to be small and far away
D. The client cannot recall anything that happened during the past 2 weeks
Answer: C
Rationale:
• Derealization involves a feeling of detachment from one's surroundings, which may include
perceptions that objects or the environment appear distorted, unreal, or unfamiliar. Option C
reflects this symptom.
54. A nurse in an acute mental health facility is planning care for a client who has dissociative
fugue. Which of the following interventions should the nurse add to the plan of care?
A. Teach the client to recognize how stress brings on a personality change in the client
B. Repeatedly present the client with information about past events
C. Make decisions for the client regarding routine daily activities
D. Work with the client on grounding techniques
Answer: D
Rationale:
• Grounding techniques can help clients with dissociative disorders, such as dissociative
fugue, reconnect with the present and their surroundings when experiencing dissociative

episodes. Teaching and practicing grounding techniques should be included in the plan of
care.
55. A nurse working in an acute mental health facility is caring for a 35-year-old female client
who has manifestations of depression. The client lives at home with her partner and two
young children. She currently smokes and has a history of chronic asthma. Which of the
following factors put the client at risk for depression? (Select all that apply)
A. Age
B. Gender
C. History of chronic asthma
D. Smoking
E. Being married
Answer: B, C, D
Rationale:
• Gender (female), history of chronic asthma, and smoking are all risk factors for depression.
Being female, having certain medical conditions like asthma, and engaging in smoking
behaviors are associated with an increased risk of developing depression.
56. A nurse working on an acute mental health unit is admitting a client who has major
depressive disorder and comorbid anxiety disorder. Which of the following actions is the
nurse's priority?
A. Placing the client on one-to-one observation
B. Assisting the client to perform ADLs
C. Encouraging the client to participate in counseling
D. Teaching the client about medication adverse effects
Answer: A
Rationale:
• Placing the client on one-to-one observation ensures the client's safety, particularly if they
are at risk of self-harm or suicide due to the depressive and anxiety symptoms.
57. A nurse working in an outpatient clinic is providing teaching to a client who has a new
diagnosis of premenstrual dysphoric disorder (PMDD). Which of the following statements by
the client indicates understanding of the teaching?
A. "I can expect my problems with PMDD to be worst when I'm menstruating."
B. "I will use light therapy 30 min a day to prevent further recurrences of PMDD."
C. "I am aware that my PMDD causes me to have rapid mood swings."

D. "I should increase my caloric intake with a nutritional supplement when my PMDD is
active."
Answer: A
Rationale:
• Premenstrual dysphoric disorder (PMDD) typically worsens in the premenstrual phase of
the menstrual cycle, so option A reflects understanding of the expected pattern of symptoms.
58. A charge nurse is discussing the care of a client who has MDD with a newly licensed
nurse. Which of the following statements by the newly licensed nurse indicates an
understanding of the teaching?
A. "Care during the continuation phase focuses on treating continued manifestations of
MDD."
B. "The treatment of MDD during the maintenance phase lasts for 6-12 weeks."
C. "The client is at greatest risk for suicide during the first weeks of an MDD episode."
D. "Medication and psychotherapy are most effective during the acute phase of MDD."
Answer: A
Rationale:
• The continuation phase of treatment for major depressive disorder (MDD) focuses on
preventing relapse by continuing antidepressant therapy and addressing residual symptoms.
59. A nurse is interviewing a 25-year-old client who has a new diagnosis of dysthymic
disorder. Which of the following findings should the nurse expect?
A. Wide fluctuations of mood
B. Report of a minimum of 5 clinical findings of depression
C. Presence of manifestations for at least 2 years
D. Inflated sense of self-esteem
Answer: C
Rationale:
• Dysthymic disorder is characterized by a chronic, low-grade depression lasting for at least 2
years, without the presence of major depressive episodes.
60. A nurse is planning care for a client who has bipolar disorder and is experiencing a manic
episode. Which of the following interventions should the nurse include in the plan of care?
(Select all that apply)
A. Provide flexible client behavior expectations
B. Offer concise explanations
C. Establish consistent limits

D. Disregard client complaints
E. Use a firm approach with communication
Answer: B, C, E
Rationale:
• During a manic episode, clients with bipolar disorder may have difficulty focusing and
maintaining attention. Therefore, providing concise explanations (option B), establishing
consistent limits (option C), and using a firm approach with communication (option E) are
important to help manage their symptoms and maintain safety. Providing flexible behavior
expectations (option A) and disregarding client complaints (option D) are not appropriate
interventions during a manic episode.
61. A nurse is teaching a newly licensed nurse about the use of ECT for the treatment of
bipolar disorder. Which of the following statements by the newly licensed nurse indicates
understanding?
A. "ECT is the recommended initial treatment for bipolar disorder."
B. "ECT is contraindicated for clients who have suicidal ideation."
C. "ECT is effective for clients who are experiencing severe mania."
D. "ECT is prescribed to prevent relapse of bipolar behavior."
Answer: C
Rationale:
• Electroconvulsive therapy (ECT) can be effective for clients experiencing severe manic or
depressive episodes in bipolar disorder, particularly when other treatments have not been
successful in managing symptoms.
62. A nurse is caring for a client who has bipolar disorder. The client states, "I am very rich,
and I feel I must give my money to you." Which of the following responses should the nurse
make?
A. "Why do you think you feel the need to give money away?"
B. "I am here to provide care and cannot accept this from you."
C. "I can request that your case manager discuss appropriate charity options with you."
D. "You should know that giving away your money is inappropriate."
Answer: B
Rationale:
• Option B establishes a therapeutic boundary, acknowledging the client's statement while
clearly stating the nurse's role and ethical boundaries regarding accepting gifts from clients.

63. A nurse in an acute mental health facility is caring for a client who has bipolar disorder.
Which of the following is the priority nursing action?
A. Set consistent limits for expected client behavior
B. Administer prescribed medications as scheduled
C. Provide the client with step-by-step instructions during hygiene activities
D. Monitor the client for escalating behavior
Answer: D
Rationale:
• Monitoring the client for escalating behavior is the priority to ensure early intervention and
prevent potential harm to the client or others during a manic or depressive episode.
64. A nurse is discussing relapse prevention with a client who has bipolar disorder. Which of
the following information should the nurse include in the teaching? (Select all that apply)
A. Use caffeine in moderation to prevent relapse
B. Difficulty sleeping can indicate a relapse
C. Begin taking your medications as soon as a relapse begins
D. Participating in psychotherapy can help prevent a relapse
E. Anhedonia is a clinical manifestation of a depressive relapse
Answer: B, C, D
Rationale:
• Difficulty sleeping can be an early indicator of a relapse in bipolar disorder, so it's important
to recognize this symptom (option B).
• Beginning medication promptly at the onset of symptoms can help prevent a full relapse and
minimize the severity of symptoms (option C).
• Psychotherapy, such as cognitive-behavioral therapy or interpersonal therapy, can be
effective in helping individuals with bipolar disorder manage symptoms and prevent relapse
(option D).
65. A nurse is caring for a client who has substance-induced psychotic disorder and is
experiencing auditory hallucinations. The client states, "The voices won't leave me alone!"
Which of the following statements should the nurse make? (Select all that apply)
A. "When did you start hearing the voices?"
B. "The voices are not real, or else we would both hear them."
C. "It must be scary to hear voices."
D. "Are the voices telling you to hurt yourself?"
E. "Why are the voices talking to only you?"

Answer: A, C, D
Rationale:
• These responses demonstrate therapeutic communication techniques, showing empathy,
exploring the client's experience, and assessing for potential risk of harm associated with the
auditory hallucinations.
66. A nurse is completing an admission assessment for a client who has schizophrenia. Which
of the following findings should the nurse document as positive symptoms? (Select all that
apply)
A. Auditory hallucination
B. Lack of motivation
C. Use of clang association
D. Delusion of persecution
E. Constantly waving arms
F. Flat affect
Answer: A, C, D, E
Rationale:
• Positive symptoms of schizophrenia are behaviors or experiences that are added to a
person's normal way of functioning and include auditory hallucinations (option A), clang
association (option C), delusions of persecution (option D), and bizarre behaviors such as
waving arms (option E).
67. A nurse is caring for a client who has schizoaffective disorder. Which of the following
statements indicates the client is experiencing depersonalization?
A. "I am a superhero and am immortal."
B. "I am no one, and everyone is me."
C. "I feel monsters pinching me all over."
D. "I know that you are stealing my thoughts."
Answer: B
Rationale:
Depersonalization involves feelings of detachment from oneself or feeling as though one's
identity is altered or nonexistent. Option B reflects a sense of identity confusion or
dissolution, indicating depersonalization.
68. A nurse is caring for a client on an acute mental health unit. The client reports hearing
voices that are telling her to "kill your doctor." Which of the following actions should the
nurse take first?

A. Use therapeutic communication to discuss the hallucination with the client
B. Initiate one-to-one observation of the client
C. Focus the client on reality
D. Notify the provider of the client's statement
Answer: B
Rationale:
The first action should be to ensure the safety of both the client and others by initiating oneto-one observation to closely monitor the client's behavior and intervene promptly if needed.
69. A nurse is speaking with a client who has schizophrenia when he suddenly seems to stop
focusing on the nurse's questions and begins looking at the ceiling and talking to himself.
Which of the following actions should the nurse take?
A. Stop the interview at this point, and resume later when the client is better able to
concentrate.
B. Ask the client, "Are you seeing something on the ceiling?"
C. Tell the client, "You seem to be looking at something on the ceiling. I see something there,
too."
D. Continue the interview without comment on the client's behavior.
Answer: B
Rationale:
Asking the client about what they are experiencing acknowledges their behavior and
demonstrates therapeutic communication while assessing for potential hallucinations.
70. A nurse manager is discussing the care of a client who has a personality disorder with a
newly licensed nurse. Which of the following statements by the newly licensed nurse
indicates an understanding of the teaching?
A. "I can promote my client's sense of control by establishing a schedule."
B. "I should encourage clients who have a schizoid personality disorder to increase
socialization."
C. "I should practice limit-setting to help prevent client manipulation."
D. "I should implement assertiveness training with clients who have antisocial personality
disorder."
Answer: C
Rationale:
Clients with personality disorders may attempt to manipulate others, so practicing limitsetting is important to maintain boundaries and promote a therapeutic environment.

71. A nurse is caring for a client who has avoidant personality disorder. Which of the
following statements is expected from a client who has this type of personality disorder?
A. "I'm scared that you're going to leave me."
B. "I'll go to group therapy if you'll let me smoke."
C. "I need to feel that everyone admires me."
D. "I sometimes feel better if I cut myself."
Answer: A
Rationale:
Avoidant personality disorder is characterized by pervasive feelings of inadequacy,
hypersensitivity to negative evaluation, and avoidance of social interaction due to fear of
rejection or criticism. Option A reflects the fear of abandonment typical in individuals with
this disorder.
72. A nurse is caring for a client who has borderline personality disorder. The client says,
"The nurse on the evening shift is always nice! You are the meanest nurse ever!" The nurse
should recognize the client's statement as an example of which of the following defense
mechanisms?
A. Regression
B. Splitting
C. Undoing
D. Identification
Answer: B
Rationale:
Splitting is a defense mechanism commonly seen in individuals with borderline personality
disorder, where they categorize people or situations as all good or all bad, without
considering the complexities or nuances. This statement reflects the client's tendency to see
people in extreme, polarized terms.
73. A nurse is assisting with a court-ordered evaluation of a client who has antisocial
personality disorder. Which of the following findings should the nurse expect? (Select all that
apply)
A. Demonstrates extreme anxiety when placed in a social situation
B. Has difficulty making even simple decisions
C. Attempts to convince other clients to give him their belongings
D. Becomes agitated if his personal area is not neat and orderly
E. Blames others for his past and current problems

Answer: C, E
Rationale:
Clients with antisocial personality disorder often exhibit manipulative behavior, such as
attempting to exploit or deceive others, including convincing others to give them their
belongings. They also commonly blame others for their own problems, refusing to accept
responsibility for their actions.
74. A charge nurse is preparing a staff education session on personality disorders. Which of
the following personality characteristics associated with all of the personality disorders
should the charge nurse include in the teaching? (Select all that apply)
A. Difficulty in getting along with other members of a group
B. Belief in the ability to become invisible during times of stress
C. Display of defense mechanisms when routines are changed
D. Claiming to be more important than other persons
E. Difficulty understanding why it is inappropriate to have a personal relationship with staff
Answer: A, D
Rationale:
Difficulty in getting along with others and a sense of entitlement or belief in being more
important than others are common characteristics seen across various personality disorders.
75. A nurse is caring for a client who has early stage Alzheimer's disease and a new
prescription for donepezil. The nurse should include which of the following statements when
teaching the client about the medication?
A. "You should avoid taking over-the-counter acetaminophen while on donepezil."
B. "You can expect the progression of cognitive decline to slow with donepezil."
C. "You will be screened for underlying kidney disease prior to starting donepezil."
D. "You should stop taking donepezil if you experience nausea or diarrhea."
Answer: B
Rationale:
Donepezil is a cholinesterase inhibitor used to slow cognitive decline in Alzheimer's disease.
Option B provides accurate information about the expected therapeutic effect of the
medication.
76. A nurse in a long-term care facility is caring for a client who has major neurocognitive
disorder and attempts to wander out of the building. The client states, "I have to get home."
Which of the following statements should the nurse make?
A. "You have forgotten that this is your home."

B. "You cannot go outside without a staff member."
C. "Why would you want to leave? Aren't you happy with your care?"
D. "I am your nurse. Let's walk together to your room."
Answer: B
Rationale:
Option B provides clear redirection and ensures the safety of the client by addressing the
wandering behavior and emphasizing the need for staff supervision.
77. A home health nurse is making a visit to a client who has Alzheimer's disease to assess the
home for safety. Which of the following suggestions should the nurse make to decrease the
client's risk for injury? (Select all that apply)
A. Install childproof door locks.
B. Place rugs over electrical cords.
C. Mark cleaning supplies with colored tape.
D. Place the client's mattress on the floor.
E. Install light fixtures above stairs.
Answer: A, C, E
Rationale:
Installing childproof door locks prevents the client from accessing hazardous areas. Marking
cleaning supplies with colored tape helps the client identify potentially harmful substances.
Installing light fixtures above stairs improves visibility and reduces the risk of falls.
78. A nurse is making a home visit to a client who is in the late stage of Alzheimer's disease.
The client's partner, who is the primary caregiver, wishes to discuss concerns about the
client's nutrition and the stress of providing care.
Which of the following actions should the nurse take?
A. Verify that a current power of attorney document is on file.
B. Instruct the client's partner to offer finger foods to increase oral intake.
C. Provide information on resources for respite care.
D. Schedules the client for placement of an enteral feeding tube.
Answer: C
Rationale:
Providing information on resources for respite care supports the caregiver by offering
temporary relief from the demands of caregiving, allowing time for self-care and reducing
caregiver stress.

79. A nurse is performing an admission assessment for a client who has delirium related to an
acute UTI. Which of the following findings should the nurse expect? (Select all that apply)
A. History of gradual memory loss
B. Family report of personality changes
C. Hallucinations
D. Unaltered level of consciousness
E. Restlessness
Answer: C, E
Rationale:
Delirium is characterized by acute changes in cognition and consciousness. Hallucinations
and restlessness are common manifestations of delirium. Gradual memory loss and
personality changes are more indicative of chronic cognitive disorders such as dementia, not
delirium.
80. A nurse is planning a staff education program on substance use in older adults. Which of
the following is appropriate for the nurse to include in the presentation?
A. Older adults require higher doses of a substance to achieve a desired effect.
B. Older adults commonly use rationalization to cope with a substance use disorder.
C. Older adults are at an increased risk for substance use following retirement.
D. Older adults develop substance use to mask manifestations of dementia.
Answer: C
Rationale:
Older adults are at an increased risk for substance use following retirement due to factors
such as social isolation, loss of identity, and boredom. Retirement can result in significant life
changes that may contribute to substance use disorders.
81. A nurse is assessing a client who has alcohol use disorder and is experiencing withdrawal.
Which of the following findings should the nurse expect? (Select all that apply)
A. Bradycardia
B. Fine tremors of both hands
C. Hypotension
D. Vomiting
E. Restlessness
Answer: B, D, E
Rationale:

Fine tremors of both hands, vomiting, and restlessness are common manifestations of alcohol
withdrawal. Bradycardia and hypotension are not typically associated with alcohol
withdrawal; rather, tachycardia and hypertension are more common findings.
82. A nurse is planning care for a client who is experiencing benzodiazepine withdrawal.
Which of the following interventions should the nurse identify as the priority?
A. Orient the client frequently to time, place, and person.
B. Offer fluids and nourishing diet as tolerated.
C. Implement seizure precautions.
D. Encourage participation in group therapy sessions.
Answer: C
Rationale:
The priority intervention for a client experiencing benzodiazepine withdrawal is to implement
seizure precautions. Benzodiazepine withdrawal can lead to seizures, so it is crucial to ensure
the client's safety by implementing measures to prevent injury during a seizure.
83. A nurse is caring for a client who has alcohol use disorder. The client is no longer
experiencing withdrawal manifestations. Which of the following medications should the
nurse anticipate administering to assist the client with maintaining abstinence from alcohol?
A. Chlordiazepoxide
B. Bupropion
C. Disulfiram
D. Carbamazepine
Answer: C
Rationale:
Disulfiram is an aversive medication used to assist clients with alcohol use disorder in
maintaining abstinence. It works by producing unpleasant effects (e.g., nausea, vomiting)
when alcohol is consumed, thus discouraging the client from drinking.
84. A nurse is providing teaching to the family of a client who has a substance use disorder.
Which of the following statements by a family member indicate an understanding of the
teaching? (Select all that apply)
A. "We need to understand that she is responsible for her disorder."
B. "Eliminating any codependent behavior will promote her recovery."
C. "She should participate in an Al-Anon group to help her recover."
D. "The primary goal of her treatment is abstinence from substance use."
E. "She needs to discuss her feelings about substance use to help her recover."

Answer: B, D
Rationale:
Understanding that eliminating codependent behavior and recognizing the primary goal of
treatment as abstinence from substance use are important aspects of supporting a family
member with a substance use disorder.
85. A nurse is preparing to obtain a nursing history from a client who has a new diagnosis of
anorexia nervosa. Which of the following questions should the nurse include in the
assessment? (Select all that apply)
A. "What is your relationship like with your family?"
B. "Why do you want to lose weight?"
C. "Would you describe your current eating habits?"
D. "At what weight do you believe you will look better?"
E. "Can you discuss your feelings about your appearance?"
Answer: B, C, D, E
Rationale:
These questions are relevant to assessing a client with anorexia nervosa and can provide
valuable information about the client's beliefs, behaviors, and feelings related to food, weight,
and body image.
86. A nurse is caring for an adolescent client who has anorexia nervosa with rapid weight loss
and a current weight of 90 lb. Which of the following statements indicates the client is
experiencing the cognitive distortion catastrophizing?
A. "Life isn't worth living if I gain weight."
B. "Don't pretend like you don't know how fat I am."
C. "If I could be skinny, I know I'd be popular."
D. "When I look in the mirror, I see myself as obese."
Answer: A
Rationale:
Catastrophizing involves the irrational belief that something is far worse than it actually is.
The statement "Life isn't worth living if I gain weight" reflects catastrophic thinking about
weight gain.
87. A nurse is performing an admission assessment of a client who has bulimia nervosa with
purging behavior. Which of the following is an expected finding? (Select all that apply)
A. Amenorrhea
B. Hypokalemia

C. Mottling of the skin
D. Slightly elevated body weight
E. Presence of lanugo on the face
Answer: B, C, E
Rationale:
Hypokalemia: Purging behaviors, such as vomiting and laxative abuse, can lead to electrolyte
imbalances, including hypokalemia, which is characterized by low potassium levels.
Mottling of the skin: Mottling of the skin, or a blotchy appearance, can result from electrolyte
imbalances and dehydration due to purging behaviors.
Presence of lanugo on the face: Lanugo, fine soft hair, may develop on the face and other
areas of the body in individuals with eating disorders due to malnutrition and the body's
attempt to conserve heat.
88. A nurse on an acute care unit is planning care for a client who has anorexia nervosa with
binge-eating and purging behavior. Which of the following nursing actions should the nurse
include in the client's plan of care?
A. Allow the client to select preferred meal times.
B. Establish consequences for purging behavior.
C. Provide the client with a high-fat diet at the start of treatment.
D. Implement one-to-one observation during meal times.
Answer: D
Rationale:
Implement one-to-one observation during meal times: Clients with anorexia nervosa with
binge-eating and purging behavior may require close supervision during meals to prevent
purging behaviors and ensure adequate nutrition intake.
89. A nurse is caring for a client who has bulimia nervosa and has stopped purging behavior.
The client tells the nurse that she is afraid she is going to gain weight. Which of the following
responses should the nurse make?
A. "Many clients are concerned about their weight. However the dietitian will ensure that you
don't get too many calories in your diet."
B. "Instead of worrying about your weight, try to focus on other problems at this time."
C. "I understand you have concerns about your weight, but first, let's talk about your recent
accomplishments."
D. "You are not overweight, and the staff will ensure that you do not gain weight while you
are in the hospital. We know that is important to you."

Answer: C
Rationale:
"I understand you have concerns about your weight, but first, let's talk about your recent
accomplishments.": This response acknowledges the client's concerns about weight while
redirecting the conversation to focus on positive achievements, which can help shift the
client's attention away from weight concerns.
90. A nurse is discussing the risk factors for somatic symptom disorder with a newly licensed
nurse. Which of the following risk factors should the nurse include? (Select all that apply)
A. Age older than 65 years
B. Anxiety disorder
C. Female gender
D. Coronary artery disease
E. Obesity
Answer: B, C, E
Rationale:
Anxiety disorder: Anxiety disorders, including generalized anxiety disorder, panic disorder,
and social anxiety disorder, are risk factors for somatic symptom disorder due to the
heightened focus on physical symptoms.
Female gender: Somatic symptom disorder is more commonly diagnosed in females than in
males.
Obesity: Obesity is a risk factor for somatic symptom disorder, possibly due to the increased
likelihood of experiencing various physical symptoms associated with obesity.
91. A nurse is reviewing the medical record of a client who has conversion disorder. Which of
the following findings should the nurse identify as placing the client at risk for conversion
disorder?
A. Death of a child 2 months ago
B. Recent weight loss of 30 lb
C. Retirement 1 year ago
D. History of migraine headaches
Answer: A
Rationale:
Death of a child 2 months ago: Traumatic or stressful life events, such as the death of a loved
one, can increase the risk of conversion disorder, which involves the manifestation of
neurological symptoms without a medical cause.

92. A nurse is assessing a client who has illness anxiety disorder. Which of the following
findings should the nurse expect? (Select all that apply)
A. Obsessive thoughts about disease
B. History of childhood abuse
C. Avoidance of health care providers
D. Depressive disorder
E. Narcissistic personality
Answer: A, C, D
Rationale:
93. A nurse is developing a plan of care for a client who has conversion disorder. Which of
the following actions should the nurse include?
A. Encourage the client to spend time alone in his room
B. Monitor the client for self-harm once per day
C. Allow the client unlimited time to discuss physical manifestations
D. Discuss alternative coping strategies with the client
Answer: D
Rationale:
Discuss alternative coping strategies with the client: Since conversion disorder involves the
manifestation of neurological symptoms without a medical cause, the nurse should focus on
helping the client develop alternative coping strategies to manage stressors or underlying
psychological issues contributing to the disorder.
93. A nurse is counseling a client who has factitious disorder imposed on another. Which of
the following client statements should the nurse expect?
A. "I had to pretend I was injured in order to get disability benefits."
B. "I know that my abdominal pain is caused by a malignant tumor."
C. "I needed to make my son sick so that someone else would take care of him for a while."
D. "I became deaf when I heard that my husband was having an affair with my best friend."
Answer: C
Rationale:
"I needed to make my son sick so that someone else would take care of him for a while.":
Factitious disorder imposed on another involves falsifying illness or injury in another person
under the individual's care, often to gain attention or fulfill emotional needs.
94. A nurse working in a mental health clinic is providing teaching to a client who has a new
prescription for diazepam for generalized anxiety disorder.

Which of the following information should the nurse provide?
A. Three to six weeks of treatment is required to achieve therapeutic benefit
B. Combining alcohol with diazepam will produce a paradoxical response
C. Diazepam has a lower risk for dependence than other antianxiety medications
D. Report confusion as a potential indication of toxicity
Answer: B
Rationale:
Combining alcohol with diazepam will produce a paradoxical response: Combining alcohol
with benzodiazepines, such as diazepam, can potentiate sedative effects and increase the risk
of respiratory depression and overdose.
95. A nurse working in an emergency department is caring for a client who has
benzodiazepine toxicity due to an overdose. Which of the following actions is the nurse's
priority?
A. Administer flumazenil
B. Identify the client's level of orientation
C. Infuse IV fluids
D. Prepare the client for gastric lavage
Answer: C
Rationale:
Infuse IV fluids: The priority in managing benzodiazepine toxicity is to maintain adequate
hydration and support vital functions. IV fluids help enhance renal excretion of the drug and
prevent dehydration.
96. A nurse is caring for a client who is to begin taking fluoxetine for treatment of generalized
anxiety disorder. Which of the following statements indicates the client understands the use
of this medication?
A. "I will take the medication at bedtime."
B. "I will follow a low-sodium diet while taking this medication."
C. "I will need to discontinue this medication slowly."
D. "I will be at risk for weight loss with long term use of this medication."
Answer: C
Rationale:
"I will need to discontinue this medication slowly.": Fluoxetine, an SSRI, should be tapered
gradually to prevent withdrawal symptoms, such as dizziness, nausea, and headache.

97. A nurse is assessing a client 4 hr after receiving an initial dose of fluoxetine. Which of the
following findings should the nurse report to the provider as indications of serotonin
syndrome? (Select all that apply)
A. Hypothermia
B. Hallucinations
C. Muscular flaccidity
D. Diaphoresis
E. Agitation
Answer: B, D, E
Rationale:
Hallucinations: Serotonin syndrome can cause hallucinations, which are perceptual
disturbances involving sensory experiences in the absence of external stimuli.
Diaphoresis: Excessive sweating or diaphoresis is a common symptom of serotonin
syndrome, reflecting the body's autonomic response to increased serotonin levels.
Agitation: Agitation is a behavioral manifestation of serotonin syndrome, characterized by
restlessness, irritability, and heightened arousal.
98. A nurse is caring for a client who takes paroxetine to treat PTSD. The client states that he
grinds his teeth during the night, which causes pain in his mouth. The nurse should identify
which of the following interventions as possible measures to manage the client's bruxism?
(Select all that apply)
A. Concurrent administration of buspirone
B. Administration of a different SSRI
C. Use of a mouth guard
D. Changing to a different class of antianxiety medication
E. Increasing the dose of paroxetine
Answer: A, C, D
Rationale:
Concurrent administration of buspirone: Buspirone, an anxiolytic medication, can help reduce
anxiety and potentially decrease bruxism symptoms.
Use of a mouth guard: A mouth guard can protect the teeth and reduce damage caused by
bruxism during sleep.
Changing to a different class of antianxiety medication: Switching to a different class of
antianxiety medication may alleviate bruxism symptoms associated with SSRIs like
paroxetine.

99. A nurse is providing teaching to a client who has a new prescription for amitriptyline.
Which of the following statements by the client indicates an understanding of the teaching?
A. "While taking this medication, I'll need to stay out of the sun to avoid a skin rash."
B. "I may feel drowsy for a few weeks after starting this medication."
C. "I cannot eat my favorite pizza with pepperoni while taking this medication."
D. "This medication will help me lose the weight that I have gained over the last year."
Answer: B
Rationale:
"I may feel drowsy for a few weeks after starting this medication.": Amitriptyline, a tricyclic
antidepressant, commonly causes sedation and drowsiness, especially during the initial weeks
of treatment.
100. A nurse is caring for a client who is taking phenelzine. For which of the following
adverse effects should the nurse monitor? (Select all that apply)
A. Elevated blood glucose level
B. Orthostatic hypotension
C. Priapism
D. Headache
E. Bruxism
Answer: B, C
Rationale:
Orthostatic hypotension: Phenelzine, a monoamine oxidase inhibitor (MAOI), can cause
orthostatic hypotension, a sudden drop in blood pressure upon standing, which can lead to
dizziness and falls.
Priapism: Priapism, a prolonged and painful erection unrelated to sexual stimulation, is a rare
but serious adverse effect of phenelzine therapy and requires immediate medical attention.
Obsessive thoughts about disease: Individuals with illness anxiety disorder often have
preoccupation with the possibility of having a serious illness despite absence of medical
evidence. This preoccupation manifests as obsessive thoughts about disease.
Avoidance of health care providers: Despite being anxious about health, individuals with
illness anxiety disorder may avoid seeking medical care out of fear of confirming their fears
or being diagnosed with a serious illness.
Depressive disorder: Depression commonly co-occurs with illness anxiety disorder. The
constant worry about health and fear of having a serious illness can lead to depressive
symptoms.

101. A nurse is reviewing the medical record of a client who has a new prescription for
bupropion for depression. Which of the following findings is the priority for the nurse to
report to the provider?
A. The client has a family history of SAD.
B. The client currently smokes 1.5 packs of cigarettes per day.
C. The client had a motor vehicle crash last year and sustained a head injury.
D. The client has a BMI of 25 and has gained 10 lb over the last year.
Answer: C
Rationale:
The client had a motor vehicle crash last year and sustained a head injury: Bupropion, an
antidepressant medication, is contraindicated in clients with a history of seizures or head
trauma due to an increased risk of seizures associated with its use.
102. A nurse is teaching a client who has a new prescription for imipramine how to minimize
anticholinergic effects. Which of the following instructions should the nurse include in the
teaching? (Select all that apply)
A. Void just before taking the medication
B. Increase the dietary intake of potassium
C. Wear sunglasses when outside
D. Change positions slowly when getting up
E. Chew sugarless gum
Answer: D, E
Rationale:
Change positions slowly when getting up: Imipramine, a tricyclic antidepressant, can cause
orthostatic hypotension, so advising the client to change positions slowly can help prevent
dizziness and falls.
Chew sugarless gum: Chewing sugarless gum can help alleviate dry mouth, a common
anticholinergic side effect associated with imipramine therapy.
103. A charge nurse is discussing mirtazapine with a newly licensed nurse. Which of the
following statements by the newly licensed nurse indicates understanding?
A. "This medication increases the release of serotonin and norepinephrine."
B. "I will need to monitor the client for hyponatremia while taking this medication."
C. "This medication is contraindicated for clients who have an eating disorder."
D. "Sexual dysfunction is a common adverse effect of this medication."
Answer: A

Rationale:
"This medication increases the release of serotonin and norepinephrine.": Mirtazapine is a
noradrenergic and specific serotonergic antidepressant (NaSSA) that increases the release of
serotonin and norepinephrine, leading to its antidepressant effects.
104. A nurse is caring for a client who is prescribed lithium therapy. The client states that he
wants to take ibuprofen for osteoarthritis pain relief. Which of the following statements
should the nurse make?
A. "That is a good choice. Ibuprofen does not interact with lithium."
B. "Regular aspirin would be a better choice than ibuprofen."
C. "Lithium decreases the effectiveness of ibuprofen."
D. "The ibuprofen will make your lithium level fall too low."
Answer: D
Rationale:
"The ibuprofen will make your lithium level fall too low.": Ibuprofen can increase the
excretion of lithium by the kidneys, leading to decreased lithium levels and potentially
reducing its therapeutic effects.
105. A nurse is discussing early indications of toxicity with a client who has a new
prescription for lithium carbonate for bipolar disorder. The nurse should include which of the
following manifestations in the teaching? (Select all that apply)
A. Constipation
B. Polyuria
C. Rash
D. Muscle weakness
E. Tinnitus
Answer: B, C, E
Rationale:
Polyuria: Lithium toxicity can manifest with increased urination (polyuria) due to its effects
on renal function.
Rash: Skin rash can occur as an early sign of lithium toxicity and should be promptly
reported to the healthcare provider.
Tinnitus: Tinnitus, or ringing in the ears, can occur with lithium toxicity and is considered an
early sign of toxicity.

106. A nurse is discussing routine follow-up needs with a client who has a new prescription
for valproate. The nurse should inform the client of the need for routine monitoring of which
of the following?
A. AST/ALT and LDH
B. Creatinine and BUN
C. WBC and granulocyte counts
D. Serum sodium and potassium
Answer: A
Rationale:
AST/ALT and LDH: Valproate, an anticonvulsant medication commonly used in the
treatment of bipolar disorder, can cause hepatotoxicity. Therefore, routine monitoring of liver
function tests, including AST, ALT, and LDH, is necessary to assess liver function and detect
potential hepatotoxicity.
107. A nurse is caring for a client who is experiencing extreme mania due to bipolar disorder.
Prior to administration of lithium carbonate, the client's lithium blood level 1.2 mEq/L.
Which of the following actions should the nurse take?
A. Administer the next dose of lithium carbonate as scheduled.
B. Prepare for administration of aminophylline.
C. Notify the provider for a possible increase in the dosage of lithium carbonate.
D. Request a stat repeat of the client's lithium blood level.
Answer: D
Rationale:
Request a stat repeat of the client's lithium blood level: A lithium blood level of 1.2 mEq/L is
within the therapeutic range for treating acute mania; however, before administering the next
dose of lithium carbonate, it is prudent to verify the accuracy of the blood level with a repeat
test, especially considering the client's extreme mania.
108. A nurse is admitting a client who has a new diagnosis of bipolar disorder and is
scheduled to begin lithium therapy. When collecting a medical history from the client's adult
daughter, which of the following statements is the priority to report to the provider?
A. "My mother has diabetes that is controlled by her diet."
B. "My mother recently completed a course of prednisone for acute bronchitis."
C. "My mother received her flu vaccine last month."
D. "My mother is currently on furosemide for her congestive heart failure."
Answer: D

Rationale:
"My mother is currently on furosemide for her congestive heart failure.": Furosemide, a loop
diuretic, can increase lithium levels by reducing its renal clearance, potentially leading to
lithium toxicity. Therefore, the client's concurrent use of furosemide warrants immediate
attention and consideration of adjusting the lithium dosage.
109. A nurse is caring for a client who has schizophrenia and exhibits a lack of grooming and
a flat affect. The nurse should anticipate a prescription of which of the following
medications?
A. Chlorpromazine
B. Thiothixene
C. Risperidone
D. Haloperidol
Answer: A
Rationale:
Chlorpromazine: Chlorpromazine is a first-generation antipsychotic medication often used to
manage symptoms of schizophrenia, including flat affect and lack of grooming. It has a
sedating effect and can help alleviate psychotic symptoms.
110. A nurse is caring for a client who takes ziprasidone. The client reports difficulty
swallowing the oral medication and becomes extremely agitated with injectable
administration. The nurse should contact the provider to discuss a change in which of the
following medications? (Select all that apply)
A. Olanzapine
B. Quetiapine
C. Aripiprazole
D. Clozapine
E. Asenapine
Answer: A, D
Rationale:
Olanzapine: Olanzapine is available in orally disintegrating tablets and intramuscular
formulations, providing alternative routes of administration for clients who have difficulty
swallowing oral medications or resist injectable administration.
Clozapine: Clozapine, an atypical antipsychotic, is available in orally disintegrating tablets
and can be administered via sublingual or buccal routes if the client has difficulty swallowing
or refuses oral medications.

111. A charge nurse is discussing manifestations of schizophrenia with a newly licensed
nurse. Which of the following manifestations should the charge nurse identify as being
effectively treated by first generation antipsychotics? (Select all that apply)
A. Auditory hallucinations
B. Withdrawal from social situations
C. Delusions of grandeur
D. Severe agitation
E. Anhedonia
Answer: A, D
Rationale:
Auditory hallucinations: First-generation antipsychotics, such as haloperidol and
chlorpromazine, are effective in treating positive symptoms of schizophrenia, including
hallucinations and delusions.
Severe agitation: First-generation antipsychotics have sedative properties and can help
manage agitation and aggression in clients with schizophrenia.
112. A nurse is assessing a client who is currently taking perphenazine. Which of the
following findings should the nurse identify as an extrapyramidal symptom (EPS)? (Select all
that apply)
A. Decreased LOC
B. Drooling
C. Involuntary arm movements
D. Urinary retention
E. Continual pacing
Answer: B, C, E
Rationale:
Drooling: Excessive salivation or drooling is a common extrapyramidal symptom (EPS)
associated with antipsychotic medications like perphenazine.
Involuntary arm movements: Extrapyramidal symptoms (EPS), including dystonia and
dyskinesia, can manifest as involuntary movements of the limbs, such as repetitive or jerky
motions.
Continual pacing: Akathisia, a type of EPS characterized by an inability to sit still and a
compulsion to move, can result in continual pacing or restlessness.

113. A nurse is providing discharge teaching for a client who has schizophrenia and a new
prescription for iloperidone. Which of the following client statements indicates understanding
of the teaching?
A. "I will be able to stop taking this medication as soon as I feel better."
B. "If I feel drowsy during the day, I will stop taking this medication and call my provider."
C. "I will be careful not to gain too much weight while taking this medication."
D. "This medication is highly addictive and must be withdrawn slowly."
Answer: C
Rationale:
"I will be careful not to gain too much weight while taking this medication.": Weight gain is a
common side effect of iloperidone, an antipsychotic medication. This client statement
demonstrates an understanding of the potential side effect and the importance of monitoring
weight during treatment.
114. A nurse is teaching the parents of a child who has autism spectrum disorder and a new
prescription for imipramine about indications of toxicity. Which of the following should the
nurse include in the teaching? (Select all that apply)
A. Seizures
B. Agitation
C. Photophobia
D. Dry mouth
E. Irregular pulse
Answer: A, B, E
Rationale:
Seizures: Imipramine, a tricyclic antidepressant, can cause seizures as a manifestation of
toxicity.
Agitation: Agitation is a possible indication of imipramine toxicity and should be monitored,
especially in children.
Irregular pulse: Imipramine toxicity can lead to cardiac dysrhythmias, including irregular
pulse, due to its effects on cardiac conduction.
115. A nurse is providing teaching to an adolescent client who has a new prescription for
clomipramine for OCD. Which of the following information should the nurse provide?
A. Eat a diet high in fiber
B. Check temperature daily
C. Take medication first thing in the morning before eating

D. Add extra calories to the diet as between-meal snacks
Answer: C
Rationale:
Take medication first thing in the morning before eating: Clomipramine, a tricyclic
antidepressant, is typically taken once daily in the morning to reduce the risk of sedation and
other side effects. Taking it before eating can also help minimize gastrointestinal discomfort.
116. A nurse is providing teaching to an adolescent client who is to begin taking atomoxetine
for ADHD. The nurse should instruct the client to monitor for which of the following adverse
effects? (Select all that apply)
A. Somnolence
B. Yellowing skin
C. Increased appetite
D. Fever
E. Malaise
Answer: A, C
Rationale:
Somnolence: Somnolence, or excessive sleepiness, is a common side effect of atomoxetine
therapy and should be monitored, especially initially or with dose adjustments.
Increased appetite: Atomoxetine can cause appetite changes, including increased appetite or
weight gain, which should be monitored during treatment.
117. A nurse is caring for a school-age child who has conduct disorder and a new prescription
for methylphenidate transdermal patches. Which of the following information should the
nurse provide about the medication?
A. Apply the patch once daily at bedtime
B. Place the patch carefully in a trash can after removal
C. Apply the transdermal patch to the anterior waist area
D. Remove the patch each day after 9 hr
Answer: C
Rationale:
Apply the transdermal patch to the anterior waist area: Methylphenidate transdermal patches
should be applied to clean, dry skin on the hip area or upper torso. The anterior waist area is a
suitable site for application.

118. A nurse is teaching a client who has intermittent explosive disorder about a new
prescription for fluoxetine. Which of the following information should the nurse provide?
(Select all that apply)
A. An adverse effect of this medication is CNS depression
B. Administer the medication in the morning
C. Monitor for weight loss while taking this medication
D. Therapeutic effects of this medication will take 1-3 weeks to fully develop.
E. This medication blocks the synaptic reuptake of serotonin in the brain.
Answer: B, D, E
Rationale:
Administer the medication in the morning: Fluoxetine, a selective serotonin reuptake
inhibitor (SSRI), is typically administered in the morning to minimize the risk of insomnia, a
common side effect.
Therapeutic effects of this medication will take 1-3 weeks to fully develop: It often takes
several weeks of treatment with fluoxetine for therapeutic effects to become evident in the
treatment of intermittent explosive disorder.
This medication blocks the synaptic reuptake of serotonin in the brain: Fluoxetine inhibits the
reuptake of serotonin, thereby increasing its availability in the brain, which contributes to its
therapeutic effects.
119. A nurse is providing teaching to a client who has alcohol use disorder and a new
prescription for carbamazepine. Which of the following information should the nurse include
in the teaching?
A. "This medication will help prevent seizures during alcohol withdrawal."
B. "Taking this medication will decrease your cravings for alcohol."
C. "This medication maintains your blood pressure at a normal level during alcohol
withdrawal."
D. "Taking this medication will improve your ability to maintain abstinence from alcohol."
Answer: A
Rationale:
"This medication will help prevent seizures during alcohol withdrawal.": Carbamazepine, an
anticonvulsant medication, is sometimes used to prevent seizures during alcohol withdrawal,
particularly in clients at risk for alcohol withdrawal seizures.

120. A nurse is assisting in the discharge planning for a client following alcohol
detoxification. The nurse should anticipate prescriptions for which of the following
medications to promote long-term abstinence from alcohol? (Select all that apply)
A. Lorazepam
B. Diazepam
C. Disulfiram
D. Naltrexone
E. Acamprosate
Answer: C, D, E
Rationale:
Disulfiram: Disulfiram is an aversive medication that produces unpleasant symptoms (such as
flushing, nausea, and vomiting) when alcohol is consumed, serving as a deterrent to drinking.
Naltrexone: Naltrexone is an opioid antagonist that reduces the pleasurable effects of alcohol,
helping to decrease cravings and relapse.
Acamprosate: Acamprosate helps maintain abstinence by reducing cravings for alcohol and
mitigating some of the neurochemical changes associated with alcohol dependence.
121. A nurse is evaluating a client's understanding of a new prescription for clonidine for the
treatment of opioid use disorder. Which of the following statements by the client indicates an
understanding of the teaching?
A. "Taking this medication will help reduce my craving for heroin."
B. "While taking this medication, I should keep a pack of sugarless gum."
C. "I can expect some diarrhea from taking this medicine."
D. "Each dose of this medication should be placed under my tongue to dissolve."
Answer: A
Rationale:
"Taking this medication will help reduce my craving for heroin.": Clonidine is sometimes
used off-label to manage withdrawal symptoms in individuals with opioid use disorder,
including reducing cravings.
122. A nurse is discussing the use of methadone with a newly licensed nurse. Which of the
following statements by the newly licensed nurse indicates an understanding of the teaching?
(Select all that apply)
A. "Methadone is a replacement for physical dependence to opioids."
B. "Methadone reduces the unpleasant effects associated with abstinence syndrome."
C. "Methadone can be used during opioid withdrawal and to maintain abstinence."

D. "Methadone increases the risk for acetaldehyde syndrome."
E. "Methadone must be prescribed and dispensed by an approved treatment center."
Answer: A, B, C, E
Rationale:
"Methadone is a replacement for physical dependence to opioids.": Methadone is a longacting opioid agonist used as a substitute for opioids to prevent withdrawal symptoms and
cravings.
"Methadone reduces the unpleasant effects associated with abstinence syndrome.":
Methadone helps manage withdrawal symptoms associated with opioid cessation.
"Methadone can be used during opioid withdrawal and to maintain abstinence.": Methadone
maintenance therapy is a treatment option for opioid use disorder that can help individuals
maintain abstinence from opioids.
"Methadone must be prescribed and dispensed by an approved treatment center.": Methadone
for opioid use disorder treatment is tightly regulated and must be administered through
approved opioid treatment programs.
123. A nurse is teaching a client who has tobacco use disorder about the use of nicotine gum.
Which of the following information should the nurse include in the teaching?
A. Chew the gum for no more than 10 min.
B. Rinse out the mouth immediately before chewing the gum.
C. Avoid eating 15 min prior to chewing the gum.
D. Use of the gum is limited to 90 days.
Answer: C
Rationale:
Avoid eating 15 min prior to chewing the gum: Nicotine gum should be chewed slowly until
a tingling sensation is felt, then "parked" between the cheek and gum to allow nicotine
absorption. Eating or drinking anything 15 minutes before or during use can decrease
absorption.
124. A nurse is caring for a client following the loss of her partner due to a terminal illness.
Identify the sequence of Engel's five stages of grief that the nurse should expect the client to
experience. (Select the stages of grief in the order of occurrence. All steps must be used.)
A. Developing awareness
B. Restitution
C. Shock and disbelief
D. Recovery

E. Resolution of the loss
Step 1: C. Shock and disbelief
Step 2: A. Developing awareness
Step 3: B. Restitution
Step 4: E. Resolution of the loss
Step 5: D. Recovery
A charge nurse is reviewing Kugler-Ross: Five Stages of Grief with a group of newly
licensed nurses. Which of the following stages should the charge nurse include in the
teaching? (Select all that apply)
A. Disequilibrium
B. Denial
C. Bargaining
D. Anger
E. Depression
Answer: B, C, D, E
Rationale:
All stages of grief according to Kübler-Ross should be included in the teaching:
B. Denial
C. Bargaining
D. Anger
E. Depression
125. A nurse is working with a client who has recently lost his mother. The nurse recognizes
that which of the following factors influence a client's grief and coping ability? (Select all
that apply)
A. Interpersonal relationships
B. Culture
C. Birth order
D. Religious beliefs
E. Prior experience with loss
Answer: A, B, D, E
Rationale:
Factors influencing grief and coping ability include:
A. Interpersonal relationships
B. Culture

D. Religious beliefs
E. Prior experience with loss
126. A nurse is discussing normal grief with a client who recently lost a child. Which of the
following statements made by the client indicates understanding? (Select all that apply)
A. "I may experience feelings of resentment."
B. "I will probably withdraw from others."
C. "I can expect to experience changes in sleep."
D. "It is possible that I will experience suicidal thoughts."
E. "It is expected that I will have a loss of self-esteem."
Answer: A, B, C, D
Rationale:
Normal grief responses may include:
A. "I may experience feelings of resentment."
B. "I will probably withdraw from others."
C. "I can expect to experience changes in sleep."
D. "It is possible that I will experience suicidal thoughts."
127. A nurse is caring for a client who lost his mother to cancer last month.
The client states, "I'd still have my mother if the doctor would have diagnosed her sooner."
Which of the following responses should the nurse make?
A. "You sound angry. Anger is a normal feeling associated with loss."
B. "I think you would feel better if you talked about your feelings with a support group."
C. "I understand just how you feel. I felt the same when my mother died."
D. "Do other members of your family also feel this way?"
Answer: A
Rationale:
"You sound angry. Anger is a normal feeling associated with loss.": This response
acknowledges the client's emotion of anger and validates it as a normal part of the grieving
process.
128. A nurse is assisting the parents of a school-age child who has oppositional defiant
disorder in identifying strategies to promote positive behavior. Which of the following is an
appropriate strategy for the nurse to recommend? (Select all that apply)
A. Allow the child to choose consequences for negative behavior
B. Use role-playing to act out unacceptable behavior
C. Develop a reward system for acceptable behavior

D. Encourage the child to participate in school sports
E. Be consistent when addressing unacceptable behavior
Answer: C, E
Rationale:
Strategies to promote positive behavior in a child with oppositional defiant disorder include:
Develop a reward system for acceptable behavior, Be consistent when addressing
unacceptable behavior
129. A nurse is performing an admission assessment on an adolescent client who has
depression. Which of the following manifestations should the nurse expect (Select all that
apply)
A. Fear of being alone
B. Substance use
C. Weight gain
D. Irritability
E. Aggressiveness
Answer: A, B, D, E
Rationale:
Fear of being alone: Adolescents with depression may experience a fear of being alone due to
feelings of isolation and sadness.
Substance use: Adolescents with depression may turn to substance use as a maladaptive
coping mechanism to alleviate their symptoms or numb their emotions.
Irritability: Irritability is a common symptom of depression in adolescents, often manifested
as a short temper, frustration, or moodiness.
Aggressiveness: Some adolescents with depression may display aggressive behavior as a
response to their emotional distress, frustration, or feelings of hopelessness.
130. A nurse is obtaining a health history from the parents of a 12-year-old client who has
conduct disorder. Which of the following findings should the nurse expect? (Select all that
apply)
A. Bullying of others
B. Threats of suicide
C. Law-breaking activities
D. Narcissistic behavior
E. Flat affect
Answer: A, C

Rationale:
Characteristics of conduct disorder may include:
Bullying of others, Law-breaking activities
131. A nurse in a pediatric clinic is caring for a preschool-age child who has a new diagnosis
of ADHD. When teaching the parent about this disorder, which of the following statements
should the nurse include in the teaching?
A. "Behaviors associated with ADHD are present prior to age 3."
B. "This disorder is characterized by argumentativeness."
C. "Below-average intellectual functioning is associated with ADHD."
D. "Because of this disorder, your child is at increased risk for injury."
Answer: D
Rationale:
"Because of this disorder, your child is at increased risk for injury.": Children with ADHD
often engage in impulsive behaviors and have difficulty paying attention, which can increase
their risk of accidents and injuries.
132. A nurse is assessing a 4-year-old child for indications of autism spectrum disorder. For
which of the following manifestations should the nurse assess?
A. Impulsive behavior
B. Repetitive counting
C. Destructiveness
D. Somatic problems
Answer: B
Rationale:
Repetitive counting: Repetitive behaviors, including counting, are common in children with
autism spectrum disorder.
133. A nurse is conducting chart reviews of multiple clients at a community mental health
facility. Which of the following events is an example of a client experiencing a maturational
crisis?
A. Rape
B. Marriage
C. Severe physical illness
D. Job loss
Answer: B
Rationale:

Marriage: Marriage is considered a maturational crisis because it involves significant life
changes and adjustments.
134. A nurse is caring for a client who is experiencing a crisis. Which of the following
medications might the provider prescribe? (Select all that apply)
A. Lithium carbonate
B. Paroxetine
C. Risperidone
D. Haloperidol
E. Lorazepam
Answer: D, E
Rationale:
In a crisis situation, medications such as antipsychotics (e.g., haloperidol) or benzodiazepines
(e.g., lorazepam) might be prescribed to manage acute symptoms of agitation, anxiety, or
psychosis.
135. A nurse is assessing a client who has major depressive disorder. The nurse should
identify which of the following client statements as an overt comment about suicide? (Select
all that apply)
A. "My family will be better off if I'm dead."
B. "The stress in my life is too much to handle."
C. "I wish my life was over."
D. "I don't feel like I can ever be happy again."
E. "If I kill myself then my problems will go away."
Answer: A, C, E
Rationale:
Overt comments about suicide directly express thoughts or intentions related to self-harm or
death.
136. A nurse is caring for a client who states, "I plan to commit suicide." Which of the
following assessments should the nurse identify as the priority?
A. Client's educational and economic background
B. Lethality of the method and availability of means
C. Quality of the client's social support
D. Client's insight into the reasons for the decision
Answer: B
Rationale:

Lethality of the method and availability of means: Assessing the lethality of the client's plan
and the availability of means to carry it out is essential for determining the level of risk and
appropriate interventions.
137. A nurse is assisting with the development of protocols to address the increasing number
of suicide attempts in the community. Which of the following interventions should the nurse
include as a primary intervention? (Select all that apply)
A. Conducting a suicide risk screening on all new clients
B. Creating a support group for family members of clients who completed suicide
C. Educating high school teens about suicide prevention
D. Initiating one-on-one observation for a client who has suicidal ideation
E. Teaching middle-school educators about warning indicators of suicide
Answer: A, C, E
Rationale:
Primary interventions focus on preventing suicide by addressing risk factors and educating
individuals and communities about suicide prevention.
138. A nurse is caring for a client who is on suicide precautions. Which of the following
interventions should the nurse include in the plan of care?
A. Assign the client to a private room
B. Document the client's behavior every hour
C. Allow the client to keep perfume in her room
D. Ensure that the client swallows medication
Answer: B
Rationale:
Document the client's behavior every hour: Regular documentation of the client's behavior
and mood is essential for monitoring changes and ensuring safety.
139. A nurse is conducting a class for a group of newly licensed nurses on caring for clients
who are at risk for suicide. Which of the following information should the nurse include in
the teaching?
A. A client's verbal threat of suicide is attention-seeking behavior
B. Interventions are ineffective for clients who really want to commit suicide
C. Using the term suicide increases the client's risk for a suicide attempt
D. A no-suicide contract decreases the client's risk for a suicide attempt
Answer: D
Rationale:

A no-suicide contract decreases the client's risk for a suicide attempt: No-suicide contracts,
also known as safety contracts, are not considered effective interventions for preventing
suicide and should not be relied upon as a sole intervention.
140. A nurse is conducting group therapy with a group of clients. Which of the following
statements made by a client is an example of aggressive communication?
A. "I wish you could not make me angry."
B. "I feel angry when you leave me."
C. "It makes me angry when you interrupt me."
D. "You'd better listen to me."
Answer: D
Rationale:
"You'd better listen to me.": This statement is aggressive and directive, conveying a sense of
threat or coercion.
141. A nurse is caring for a client who is speaking in a loud voice with clenched fists. Which
of the following actions should the nurse take?
A. Insist that the client stop yelling
B. Request that other staff members remain close by
C. Move as close to the client as possible
D. Walk away from the client
Answer: B
Rationale:
Request that other staff members remain close by: This option ensures the safety of both the
client and staff by having additional support available if the situation escalates.
143. A nurse is assessing a client in an inpatient mental health unit. Which of the following
findings should the nurse expect if the client is in the preassaultive stage of violence? (Select
all that apply)
A. Lethargy
B. Defensive responses to questions
C. Disorientation
D. Facial grimacing
E. Agitation
Answer: B, D, E
Rationale:

Clients in the preassaultive stage may exhibit defensive responses to questions, facial
grimacing, and increased agitation as they approach a state of heightened arousal before a
potential violent outburst.
144. A nurse is caring for a client in an inpatient mental health facility who gets up from a
chair and throws it across the day room. Which of the following is the priority nursing
action?
A. Encourage the client to express her feelings
B. Maintain eye contact with the client
C. Move the client away from others
D. Tell the client that the behavior is not acceptable
Answer: C
Rationale:
Move the client away from others: The priority is to ensure the safety of other clients and
staff by removing the client from a potentially dangerous situation.
145. A nurse is caring for a client who is screaming at staff members and other clients. Which
of the following is a therapeutic response by the nurse to the client?
A. "Stop screaming, and walk with me outside."
B. "Why are you so angry and screaming at everyone?"
C. "You will not get your way by screaming."
D. "What was going through your mind when you started screaming?"
Answer: D
Rationale:
"What was going through your mind when you started screaming?": This response
encourages the client to explore their thoughts and feelings, promoting insight and
understanding of their behavior.
146. A charge nurse is leading a peer group discussion about family and community violence.
Which of the following statements by a member of the group indicates an understanding of
the teaching?
A. "Children older than 3 are at greater risk for abuse."
B. "Substance use disorder does not increase the risk for violence."
C. "Entering an intimate relationship increases the risk for violence."
D. "Pregnancy increases the risk for violence toward the intimate partner."
Answer: D
Rationale:

"Pregnancy increases the risk for violence toward the intimate partner.": Pregnancy can
exacerbate existing intimate partner violence or increase the risk of violence in previously
nonviolent relationships.
147. A nurse is preparing to assess an infant who has shaken baby syndrome. Which of the
following is an expected finding? (Select all that apply)
A. Sunken fontanels
B. Respiratory distress
C. Retinal hemorrhage
D. Altered LOC
E. Increase in head circumference
Answer: B, C, D
Rationale:
Shaken baby syndrome can result in respiratory distress, retinal hemorrhage, and altered level
of consciousness due to the traumatic brain injury inflicted on the infant.
148. A nurse working in an emergency department is assessing a preschool-age child who
reports abdominal pain. When conducting a head-to-toe assessment, which of the following
findings should alert the nurse to possible abuse?
A. Abrasions on knees
B. Round burn marks on forearms
C. Mismatched clothing
D. Abdominal rebound tenderness
E. Areas of ecchymosis on torso
Answer: E
Rationale:
Areas of ecchymosis on torso: Ecchymosis (bruising) in areas not commonly injured
accidentally, such as the torso, can be indicative of physical abuse.
149. A nurse is preparing a community education seminar about family violence.
When discussing types of violence, the nurse should include which of the following?
A. Refusing to pay bills for a dependent, even when funds are available, is neglect.
B. Intentionally causing an older adult to fall is an example of physical violence.
C. Striking an intimate partner is an example of sexual violence.
D. Failure to provide a stimulating environment for normal development is emotional abuse.
Answer: A
Rationale:

Refusing to pay bills for a dependent, even when funds are available, is neglect: Neglect
involves the failure to provide for a dependent's basic needs, including food, shelter, medical
care, and education.
150. A nurse is caring for an adult client who has injuries resulting from intimate partner
abuse. The client does not wish to report the violence to law enforcement authorities. Which
of the following nursing actions is the highest priority?
A. Advise the client about the location of women's shelters
B. Encourage the client to participate in a support group for survivors of abuse
C. Implement case management to coordinate community and social services
D. Educate the client about the use of stress management techniques
Answer: C
Rationale:
Implement case management to coordinate community and social services: The priority is to
ensure the client's safety and provide resources for support and assistance, which can be
facilitated through case management services.
151. A nurse is discussing silent rape reaction with a newly licensed nurse.
The nurse should identify which of the following characteristics as expected for this type of
reaction? (Select all that apply)
A. Sudden development of phobias
B. Development of substance use disorder
C. Increased level of anxiety during interview
D. Reactivation of a prior physical disorder
E. Unwillingness to discuss the sexual assault
Answer: A, E
Rationale:
Sudden development of phobias: Silent rape reaction can manifest as the sudden development
of phobias, particularly related to situations or environments that trigger memories of the
assault.
E. Unwillingness to discuss the sexual assault: Individuals experiencing silent rape reaction
may be unwilling to discuss the assault due to feelings of shame, guilt, or fear of judgment.
152. A nurse is assessing a client who experienced sexual assault. Which of the following
findings indicate the client is experiencing an emotional reaction of rape-trauma syndrome?
(Select all that apply)
A. Genitourinary soreness

B. Difficulties with low self-esteem
C. Sleep disturbances
D. Emotional outburst
E. Difficulty making decisions
Answer: B, C, D, E
Rationale:
Difficulties with low self-esteem: Low self-esteem is a common emotional reaction of rapetrauma syndrome, resulting from feelings of shame, guilt, and diminished self-worth.
Sleep disturbances: Sleep disturbances, including nightmares, insomnia, or frequent waking,
are common emotional reactions to sexual assault.
Emotional outburst: Emotional outbursts, such as crying spells, anger outbursts, or mood
swings, can occur as part of the emotional reaction to rape-trauma syndrome.
Difficulty making decisions: Difficulty making decisions or concentrating is a cognitive
manifestation of rape-trauma syndrome, often related to the psychological distress
experienced by the survivor.
153. A nurse is discussing the care of a client following a sexual assault with a newly licensed
nurse. Which of the following statements by the newly licensed nurse indicates an
understanding of the teaching?
A. "I will administer prophylactic treatment for sexually transmitted infections."
B. "I am not required to obtain informed consent before the sexual assault nurse examiner
collects forensic evidence."
C. "I can expect manifestations of rape-trauma syndrome to be similar to bipolar disorder."
D. "I should use narrative documentation when documenting subjective data."
Answer: A
Rationale:
"I will administer prophylactic treatment for sexually transmitted infections.": Administering
prophylactic treatment for sexually transmitted infections is a crucial aspect of the care
provided to a sexual assault survivor to prevent potential infections.
154. A nurse is caring for a client who was recently raped. The client states, "I never should
have been out on the street alone at night." Which of the following responses should the nurse
make?
A. "Your actions had nothing to do with what happened."
B. "You should focus on recovery rather than blaming yourself for what happened."
C. "You believe this wouldn't have happened if you hadn't been out alone?"

D. "Why do you feel that you should not have been alone on the street at night?"
Answer: D
Rationale:
"Why do you feel that you should not have been alone on the street at night?": This response
acknowledges the client's statement and encourages further exploration of their feelings and
beliefs without invalidating their experience.
155. A community health nurse is leading a discussion about rape with a neighborhood task
force. Which of the following statements by a neighborhood citizen indicates an
understanding of the teaching?
A. "Rape is a crime of passion."
B. "Acquaintance rape often involves alcohol."
C. "Young adults are the typical victims of sexual assault."
D. "The majority of rapists are unknown to the victims."
Answer: B
Rationale:
"Acquaintance rape often involves alcohol.": This statement reflects an understanding of the
association between alcohol consumption and acquaintance rape, highlighting a common risk
factor in such cases.

Document Details

  • Subject: Nursing
  • Exam Authority: ATI
  • Semester/Year: 2023

Related Documents

Close

Send listing report

highlight_off

You already reported this listing

The report is private and won't be shared with the owner

rotate_right
Close
rotate_right
Close

Send Message

image
Close

My favorites

image
Close

Application Form

image
Notifications visibility rotate_right Clear all Close close
image
image
arrow_left
arrow_right